[ /tv/ /rf/ /vg/ /a/ /b/ /u/ /bo/ /fur/ /to/ /dt/ /cp/ /oe/ /bg/ /ve/ /r/ /mad/ /d/ /mu/ /cr/ /di/ /sw/ /hr/ /wh/ /lor/ /s/ /hau/ /slow/ /gf/ /vn/ /w/ /ma/ /azu/ /wn/ ] [ Main | Settings | Bookmarks | Music Player ]

No.133509 Reply
File: ОП-пик.jpg
Jpg, 105.74 KB, 800×600 - Click the image to expand
edit Find source with google Find source with iqdb
ОП-пик.jpg
Наша кафедра продолжает свою работу. Здесь мы обсуждаем математику, а также иногда отвечаем на простые вопросы доброанонов.
Тред обучения математике: >>124265

Калькуляторы:
http://online-integral.ru/
http://www.wolframalpha.com/

Предыдущий: >>130172
>> No.133512 Reply
>>133360
А если разложить в ряд в окрестности 0?
>> No.133522 Reply
Надо будет в январе сдавать зачет "от предела до интеграла". Какой путь оптимальнее: учить всю соответствующую теорию или тупо выучить 3.5 приемчика? Просто вычислительная часть матана меня вообще не интересует.
>> No.133523 Reply
>>133522
Я сам первокур, но мне помогает сначала выучить приемчики, а потом теорию, чтоб понять почему они именно такие.
>> No.133532 Reply
Помогите школьнику на экстернате. Через пару дней сдавать экзамен, в котором будет задача такого типа:
При каких значениях параметра а функция kx^3-mx+c=a имеет n корней? Например:
2/3x^3-2x+1=a
При каких значениях а уравнение имеет менее трех корней?
>> No.133533 Reply
>>133532
Основная теорема алгебры - многочлен степени n над полем комплексных чисел имеет ровно n корней.
2/3x^3-2x+1=a имеет три корня.
>> No.133539 Reply
Анон, я параноик и боюсь отчисления даже когда у меня все довольно хорошо. Можно ли в случае отчисления снова поступить в тот же универ, на ту же специальность, на бюджет второй раз?
>> No.133541 Reply
>>133539
Да, конечно. Поступать на бюджет ты можешь столько раз, сколько хочешь. Право на высшее образование ты теряешь только тогда, когда получишь диплом; пока диплома нет, поступай хоть десять раз. Однако при отчислении теряется отсрочка, её дают только один раз, при втором и последующих поступлениях отсрочки уже не будет. Кроме того, для поступления требуется наличие действующих результатов ЕГЭ, но это очевидно. Я перепоступал, я знаю.
>> No.133546 Reply
>>133533
За доказательство это не примут.
>> No.133548 Reply
репост из далёкого /б/
(>>/b/3395465)
> B ∖ ((A ∪ B) ∖ A) = {c ∣ ((c ∈ A) ∧ (c ∉ B)) ∨ ((c ∉ A) ∧ (c ∈ B))};
> Вот здесь у тебя справа на самом деле написано (A\B)∪(B\A). Это называется "симметрическая разность": А без Б, объединенное с Б без А. Эта операция обозначается треугольничком, похожим на большую букву дельта.
Я бы хотел остановиться на этом чуть подробнее. Возьмём два простеньких множества и посчитаем по шагам:
A = {1, 2}; B = {2, 3};
C = A ∪ B = {1, 2, 3};
D = C \ A = {3};
I = B \ D = {2};
И действительно, A ∩ B = {2}; Выглядит это, по крайней мере, вполне себе как пересечение.
> Непонятно вот, о чём именно ты сожалеешь и почему сожалеешь.
Это для ради немного разбавить моё унылое повествование :3 Больше не буду.
> 15а
> И ещё подсказка - вспомни, что если a<1 и b<1, то a+b < 2.
Возможно я всё же не к месту воспользовался этим новым понятием мощности, но ведь смысл к этому a<1, b<1, a+b < 2 и сводился. Говоря проще, если ни одна фигура не накладывается на другую фигуру, то объединение площадей-множеств (пусть это будут множества элементов площади, те же самые попугаи) даст больше элементов чем содержится в площади-множестве описывающей фигуры и либо вписанные фигуры вписанными не являются, либо всё же они вписаны и накладываются друг на друга, так что множества, представляющие их площади, будут пересекаться.
>> No.133555 Reply
>>133548
Я там жестоко протупил. Новый год надвигается, я проверил так много листочков, что ресурсы мозга израсходовались полностью. Хотя это и не извиняет меня, конечно. Посмотри замечание анона ниже, пожалуйста. Буду помнить об этом фейле очень долго, хотя сейчас ещё и не таких ошибок понаделаю, скорее всего.
> Я бы хотел остановиться на этом чуть подробнее
x ∈ B ∖ ((A ∪ B) ∖ A) <=>
x ∈ B ∧ x ∉ ((A ∪ B) ∖ A) <=>
x ∈ B ∧ ¬(x ∈ ((A ∪ B)∖A)) <=>
x ∈ B ∧ ¬(x ∈ (A ∪ B) ∧ x ∉ A) <=>
x ∈ B ∧ ¬(x ∈ (A ∪ B) ∧ ¬ (x ∈ A)) <=>
x ∈ B ∧ ¬((x ∈ A ∨ x ∈ B) ∧ ¬ (x ∈ A)) <=>
x ∈ B ∧ (¬(x ∈ A ∨ x ∈ B) ∨ (x ∈ A)) <=>
x ∈ B ∧ ( ((¬ x ∈ A) ∧ ¬ (x ∈ B)) ∨ (x ∈ A)) <=>
x ∈ B ∧ ( (x ∉ A ∧ x ∉ B) ∨ (x ∈ A)) <=>
x ∈ B ∧ x ∉ A ∧ x ∉ B ∨ x ∈ B ∧ x ∈ A <=>
x ∈ B ∧ x ∈ A <=>
x ∈ B ⋂ A
Эм ай вронг? Тут использовал законы де Моргана, ¬(P∧Q) = ¬P∨¬Q ; ¬(P∨Q) = ¬P ∧ ¬Q. Они, вроде как, были в книжке, которую я просил прочитать. Должны быть.
А вот к правой части придирки остались.
((c ∈ A) ∧ (c ∉ B)) ∨ ((c ∉ A) ∧ (c ∈ B)) <=>
((c ∈ A) ∧ (c ∉ B)) ∨ ((c ∈ B) ∧ (c ∉ A)) <=>
(c ∈ A\B) ∨ (c ∈ B\A) <=>
c ∈ (A\B) ∪ (B\A).
Так, нет?

Просто если x принадлежит пересечению A и B, то и c∈A∧c∉B есть ложь, и c∉A∧c∈B есть ложь.
> то объединение площадей-множеств
Оно прямо вот так и называется же, площадь пересечения фигур.
> так что множества, представляющие их площади, будут пересекаться
Проблема же не в том, пересекаются ли фигуры. Понятно, что пересекаются. Проблема в свойствах площади пересечения - нужно доказать, что она не меньше чего-то там.
>> No.133575 Reply
Здравствуй, доброанон-математик. Я не знаю про математику ничего, но есть кое-что, что меня волнует. Я могу вычислить синус или косинус произвольной фигуры, пользуясь таблицей углов. Как-то раз я решал задачу с помощью компьютерной программы, где применялась функция вычисления синуса и косинуса из угла в радианах.

Сегодня я потратил изрядно времени на поиск формулы, которая позволяет вычислить синус и косинус из любого угла, без всяких таблиц. Нашел примерно следующее "Пользуйтесь инженерным калькулятором". Анон, можешь объяснить мне, почему так? Возможно, я где-то не там искал?
Да, мои знания очень поверхностны и я не претендую на математику даже как хобби. Надеюсь в скором будущем это исправить.
>> No.133577 Reply
File: 1024px-Trigonometric_function.png
Png, 169.36 KB, 1024×1009 - Click the image to expand
edit Find source with google Find source with iqdb
1024px-Trigonometric_function.png
>> No.133582 Reply
>>133575
Нужная тебе формула называется "разложение в ряд Тейлора". Поскольку синус - это, вообще, трансцендентная функция, точное значение ты не сможешь получить. Но ты можешь получить столько верных цифр, сколько тебе нужно.

sin x = x - x^3/3! + x^5/5! - x^7/7! + ...
ruwiki://Ряд_Тейлора
Количество верных цифр выясняется через оценку остаточного члена. x - угол в радианах. Прежде чем вычислять синус, целесообразно пользоваться формулами приведения.
>> No.133585 Reply
File: Форман 8.jpg
Jpg, 14.74 KB, 309×416 - Click the image to expand
edit Find source with google Find source with iqdb
Форман 8.jpg
>>133582
> ruwiki
>> No.133586 Reply
File: 1412785722-0499580a34099b47d302e5f7079c1ab2.JPG
Jpg, 126.82 KB, 577×1024 - Click the image to expand
edit Find source with google Find source with iqdb
1412785722-0499580a34099b47d302e5f7079c1ab2.JPG
>>133585
Раскрой мысль.
>> No.133588 Reply
>>133586 как нихуя не понять. про естественный предел применимости ряда ни слова, например. посмотри английскую версию для сравнения
>> No.133589 Reply
>>133588
Теорема Тейлора, указанная в русской вики, верна. Разницы между английской и русской вики не видно.
> естественный предел применимости ряда
Что ты понимаешь под этим?
>> No.133590 Reply
>>133589 спрашивали о вычислениях, а не о доказательстве
> Что ты понимаешь под этим?
применимость ряда для вычисления значения функции
>> No.133592 Reply
>>133590
> применимость ряда для вычисления значения функции
В русской вики указано несколько формул для остаточного члена, что очень важно именно для вычислений. В английской их нет, и вообще та статья неинформативна, несмотря на изобилие примеров.
>> No.133599 Reply
>>133592 для вычислений в первую очередь важно знать при каких значениях аргумента ряд расходится. почему ряд для функции 1/(1-x^2) применим только при |x|<1, а ряд для tg(x) только при |x|<pi/2 ?
> В английской их нет
> Taylor's theorem gives quantitative estimates on the error in this approximation.
в английской есть отдельная статья, посвященная именно теореме Тейлора. а статья про ряд Тейлора рассказывает о том, как и для чего использовать ряд
не понимаю, для кого писалась русская версия: люди в теме это и так знают, а которые не в теме, те ничего не поймут
>> No.133604 Reply
>>133590
Теорема описывает условия применимости. Не понимаю, о чём ты говоришь.
>> No.133613 Reply
File: 4у.png
Png, 137.34 KB, 629×569 - Click the image to expand
edit Find source with google Find source with iqdb
4у.png
Не понимаю, что это за такой номер N, зависящий от окрестности точки А? И как понять точность "эпсилон>0" Не могу найти пояснение, что это за эпсилон такое? В других учебниках определения такие же "размытые" .

Только не надо писать, что я толстый тролль. Я серьезно не понимаю.
>> No.133614 Reply
Объясните, каким образом можно возвести число 123 в 64-ю степень, используя только 3 операции умножения.
>> No.133616 Reply
>>133613
Это какая-то функция из множества окрестностей во множество натуральных чисел.
Эпсилон - произвольное число. Идея в том, что если две величины отличаются меньше, чем на любое заданное число, то они равны. Покури философию бесконечного, лол. ruwiki://Анализ_бесконечно_малых
>> No.133617 Reply
>>133614
> каким образом можно возвести число 123 в 64-ю степень, используя только 3 операции умножения
Никаким?
>> No.133618 Reply
>>133614
Никаким.
>> No.133620 Reply
>>133614
Может не 3, а 6?
>> No.133626 Reply
File: 3кпрт.png
Png, 7.91 KB, 488×320 - Click the image to expand
edit Find source with google Find source with iqdb
3кпрт.png
Вот по этой картинке получается, что предел - это просто значение Х функции, которому соответствует значение У функции. Но тогда функция имеет много пределов, а не один, верно? Это я уже полез в зарубежные учебники на ангельском, но там как под копирку то же самое, что и в наших, но с картинками.
>> No.133630 Reply
>>133626
> предел - это просто значение Х функции, которому соответствует значение У функции
Если функция непрерывна, то да.
> Но тогда функция имеет много пределов, а не один, верно?
Не бывает просто "предела", он рассматривается в точке. А если в точке предел существует, то он единственный.
>> No.133647 Reply
File: 8ztZL0iLYCI.jpg
Jpg, 32.87 KB, 800×600 - Click the image to expand
edit Find source with google Find source with iqdb
8ztZL0iLYCI.jpg
Доброанон, поясни мне по хардкору.
>> No.133648 Reply
File: tumblr_inline_nfbar3Ezhw1rt1b52.gif
Gif, 821.75 KB, 450×253 - Click the image to expand
edit Find source with google Find source with iqdb
tumblr_inline_nfbar3Ezhw1rt1b52.gif
>>133647
Там просто 16 будет. Никаких подъебов.
>> No.133651 Reply
Да всё просто.
Действие 1: вычисляем скобки 2*2=4
Действие 2: умножение и деление имеют одинаковый приоритет, деление идёт первым 20/5=4
Действие 3: умножение 4*4=16
Вот и всё. Для самых маленьких:http://math-prosto.ru/?page=pages/order_of_action/order_of_action.php
>> No.133654 Reply
>>133651
Но ведь между 5 и ( нет знака умножения! Как так то?!
>> No.133655 Reply
>>133654
> Но ведь между 5 и ( нет знака умножения!
Это ничего не значит.
> Как так то?!
Что как так?
>> No.133656 Reply
File: Новый-точечный-рисунок-(2).png
Png, 17.46 KB, 468×337 - Click the image to expand
edit Find source with google Find source with iqdb
Новый-точечный-рисунок-(2).png
Как получилось то, что внизу картинки?
>> No.133657 Reply
>>133656
Теорема. Если p и q - корни квадратного уравнения ax^2 + bx + c = 0, то это уравнение можно разложить на множители следующим образом:
ax^2 + bx + c = a(x-p)(x-q).
>> No.133658 Reply
File: я все понял.jpg
Jpg, 164.85 KB, 400×359 - Click the image to expand
edit Find source with google Find source with iqdb
я все понял.jpg
>>133657
Спасибо.
>> No.133659 Reply
>>133655
Вот именно, знака нет, значит умножение не подразумевается. Либо его пропустили специально, чтобы добавить порядок действий.
>> No.133661 Reply
File: 13041096477117.jpg
Jpg, 69.07 KB, 600×450 - Click the image to expand
edit Find source with google Find source with iqdb
13041096477117.jpg
>>133659
> знака нет, значит умножение не подразумевается
Наркоман штоле? Как раз подразумевается.
>> No.133664 Reply
>>133661
Вовсе нет.
>> No.133666 Reply
File: cat-breathing.jpg
Jpg, 32.96 KB, 600×553 - Click the image to expand
edit Find source with google Find source with iqdb
cat-breathing.jpg
>> No.133671 Reply
>>133626
>>133630
А зачем нужны эти красная и зеленая линии?
>> No.133673 Reply
File: график.png
Png, 11.57 KB, 800×600 - Click the image to expand
edit Find source with google Find source with iqdb
график.png
>>133671
Окрестностью точки x называется какой-то интервал, содержащий точку x. пусть пока так
Выколотой окрестностью точки x называется окрестность точки x, из которой исключили саму точку x.
Образ множества - это множество образов всех его точек.

Число A называется пределом функции f в точке a, если в любой окрестности точки A лежит образ некоторой выколотой окрестности точки a.

Предел функции в точке может быть не равен значению функции в этой точке. Функция может быть даже не определена в некоторой точке, но иметь в этой точке предел. Когда мы делаем в точке предельный переход, мы рассматриваем только то, что творится вокруг точки, но не саму точку.

Пикрелейтед. Есть функция f. В точке x она ведёт себя интересно. Если бы она была непрерывной в этой точке, то её значение было бы y. Но она не непрерывна в этой точке, f(x) = y'. То есть предел функции f в точке x равен y, хотя значение функции f в точке x равно y'.

Если снять в том определении требование выколотости, получим определение непрерывной в точке функции. Предел функции в точке, в которой эта функция непрерывна, равен значению функции в этой точке.

Образно говоря, предел функции - это такая точка, которая сделала бы функцию непрерывной, а предельный переход - это переход от данной функции к непрерывной функции с такими же свойствами. Делая предельный переход, мы можем изучить непрерывную функцию и затем, гм, перенести полученные свойства на исходную функцию.

Штрих в том определении не нужно путать с символом производной. Это просто штрих, чтобы различить два числа.
Значения функции нужно смотреть на оси ординат (вертикальная линия со стрелочкой). Точки y и y' лежат на оси ординат.
Точка x - это точка на оси абсцисс (горизонтальная линия со стрелочкой).
Чёрная жирная точка над точкой x - это просто кусочек графика функции.
>> No.133675 Reply
>>133673
Непонятно, что такое образ точки
> Если бы она была непрерывной в этой точке, то её значение было бы y. Но она не непрерывна в этой точке, f(x) = y'. То есть предел функции f в точке x равен y, хотя значение функции f в точке x равно.
Так ведь все наоборот. По графику Функция нигде не прерывается, значению х соответствует у без штриха. Впервые вижу кусок графика функции где-то в стороне от неё.
>> No.133690 Reply
>>133509
Господи, ОП-пик божественнен! Формула Стокса 11 из 10!
>> No.133695 Reply
>>133675
> что такое образ точки
Значение функции в этой точке. Если y = f(x), то y называется образом точки x при отображении f, x называется прообразом точки y при отображении f.
> По графику Функция нигде не прерывается
Смотри внимательнее. Где пунктирные линии пересекаются, там дырка.
>> No.133722 Reply
File: 5f19ECbOXsk.jpg
Jpg, 30.22 KB, 604×377 - Click the image to expand
edit Find source with google Find source with iqdb
5f19ECbOXsk.jpg
> Смысл определения предела заключается в том, что какой бы мы ни взяли
длину интервала, которому принадлежит предел последовательности, всё равно, начиная с некоторого номера N все элементы последовательности будут
находиться внутри этого интервала.
Что-то бред какой-то. Для примера, берем последовательность (2n-1)/n, предел тут равен двум. Беру предел, черчу его окрестность, радиусом R=0.5 для примера, тогда часть элементов в неё не попадет
>>133695
Теперь понятно
>> No.133751 Reply
>>133722
> тогда часть элементов в неё не попадет
Элементов, которые не попадают, не бесконечно много. Лишь конечное число элементов не попадают в окрестность.
>> No.133777 Reply
Не понимаю комплексные числа. Вот f(z) - функция в четырехмерной плоскости?
>> No.133811 Reply
Уважаемые математики! Заранее извиняюсь за глупый вопрос. Чему равно произведение бесконечно малого и бесконечно большого, если оба они одного порядка?
>> No.133812 Reply
>>133777
Почему четырёхмерной? Функция ℂ→ℂ - это функция ℝ^2→ℝ^2. То есть имеются две обыкновеннейшие плоскости, и каждой точке одной плоскости сопоставлена какая-то точка другой плоскости.
ℝ - это прямая.
ℂ - это плоскость.

>>133811
Что значит "одного порядка"? Обычно "порядок малости" вводится только для бесконечно-малых.
У тебя стандартная неопределённость 0 ∙ ∞, может быть что угодно.
>> No.133813 Reply
>>133811
> Чему равно произведение бесконечно малого и бесконечно большого
Неопределенности, наверное. Попробуй посчитать предел этого произведения.
>> No.133814 Reply
File: limes.gif
Gif, 11.30 KB, 1833×208 - Click the image to expand
edit Find source with google Find source with iqdb
limes.gif
>>133812
>>133813
Или я чего-то не понимаю?
>> No.133815 Reply
File: неопред.png
Png, 102.42 KB, 1167×606 - Click the image to expand
edit Find source with google Find source with iqdb
неопред.png
>> No.133816 Reply
>>133815
Это то же самое, что и >>133814, только другими словами.
Ладно, спасибо.
>> No.133828 Reply
File: 5аууц.png
Png, 113.90 KB, 1178×438 - Click the image to expand
edit Find source with google Find source with iqdb
5аууц.png
>> No.133865 Reply
>>133555
> Они, вроде как, были в книжке, которую я просил прочитать.
Именно так. Они там есть.

(>>/b/3395582)
> Задача 1. Старейший математик среди шахматистов и старейший шахматист среди математиков — это один или тот же человек или (возможно) разные?
> Задача 2. Лучший математик среди шахматистов и лучший шахматист среди математиков — это один или тот же человек или (возможно) разные?
В принципе, мне кажется, обе эти задачи имею одинаковое решение. Пусть есть множество математиков А и множество шахматистов В, элементы этих множеств обладают некоторым свойством - возраст и успешность (назовём условно так свойство, оцениваемое по шкале лучший-худший). Возможно, во множество элементов с данным признаком (все лучшие математики/шахматисты или все старейшие математики/шахматисты) входят элементы из множества А и из множества В, а могут входить только а ∈ A или b ∈ B. Иными словами, ничто не мешает старейшему математику быть ещё и старейшим шахматистом, если он увлекается шахматами и равно ничто не мешает лучшему математику быть лучшим шахматистом. Возможно и обратное: старейший шахматист, он же старейший математик и лучший шахматист - он же лучший математик.
> Задача 3. Каждый десятый математик — шахматист, а каждый шестой шахматист — математик. Кого больше — математиков или шахматистов — и во сколько раз?
Подсказка. Сине-красное - это красно-синее.
Думаю, это непосредственным образом зависит от меры множества. Само определение "каждый энный" ничего не говорит нам об абсолютном числе математиков или шахматистов, если мы не знаем, сколько вообще этих математиков и шахматистов есть и кого из них больше.
> Задача 4. Придумать конечное семейство множеств, в котором индексами является не числовое множество, а множество слов. Записать это семейство списком, как в примере 1.
Пусть у нас будет семейство множеств A овощефруктов, а индекс i пробегает значения и множества I {i ∈ I | P(i) = зелёный, красный, оранжевый, жёлтый}:
A(зелёный) = {огурец, арбуз, капуста, яблоко, перец};
A(красный) = {перец, томат, яблоко};
A(оранжевый) = {апельсин, мандарин, морковь};
A(жёлтый) = {банан, яблоко, перец}.
> Задача 5. Написать объединение и пересечение семейства, которое вы придумали в задаче 4.
Объединением множеств A(i) будет множество A = {огурец, арбуз, капуста, яблоко, перец, томат, апельсин, мандарин, морковь, банан}.
Пересечением множеств A(i) будет пустое множество ∅, т.к. ¬(∃a)P((а ∈ A(красный)) ∧ (а ∈ A(зелёный)) ∧ (а ∈ A(оранжевый)) ∧ (а ∈ A(жёлтый)).
> Дизъюнктивное семейство множеств - это такое семейство множеств, что элементы в нём попарно не пересекаются. Точнее, если i1 ≠ i2, то Ai1 ∩ Ai2 = ∅.
> Задача 6. Придумать дизъюнктивное семейство множеств.
Пусть {i ∈ I | P(i) = 1, 2, 3, 4, 5}, тогда:
A(1) = {1, 2}; A(2) = {3, 4}; A(3) = {5, 6};
A(4) = {7, 8}; A(5) = {9, 10}
и A - дизъюнктивное семейство множеств, т.к. A(x) ∩ A(y) = ∅, где x, y ∈ I и x ≠ y. Вроде так, если я правильно понял суть. Хотя она кажется мне несколько странной. Проще говоря, дизъюнктивное семейство - это множество уникальных множеств, где ∀a ∈ A, a ∉ B, C, ..., а A, B, C ... в свою очередь принадлежат одному семейству.
> Задача 7. Предложите какое-нибудь множество такое, что семейство, которое вы придумали в задаче 6, является его покрытием.
{a ∈ A | (a ⩾ 1) ∧ (a ⩽ 10)}, ЕЯПП формулировку.
> Задача 8. Предложите какое-нибудь разбиение множества из задачи 7.
Разбиением множества А из 7 будет множество из 6. Кстати, как-то так получается, что разбиение в данном случае является и покрытием. Это нормально вообще?
>> No.133868 Reply
>>133865
> Объединением множеств A(i) будет множество A = {огурец, арбуз, капуста, яблоко, перец, томат, апельсин, мандарин, морковь, банан}.
Объединением семейств ...
> Пересечением множеств A(i) будет пустое множество ∅, т.к. ¬(∃a)P((а ∈ A(красный)) ∧ (а ∈ A(зелёный)) ∧ (а ∈ A(оранжевый)) ∧ (а ∈ A(жёлтый)).
Пересечением семейств ...
> Разбиением множества А из 7 будет множество из 6.
... семейство множеств из 6.
молниеносный быстрофикс
>> No.133869 Reply
>>133865
Эти задачи на понятия пересечения и объединения.
Моя не понял, так как ты ответишь на задачи 1 и 2? Там либо один и тот же, либо не всегда один и тот же.
Задача 3. Подумай ещё. Количество элементов во множестве - мощность. Мощность есть у каждого множества, а вот измеримым является-таки не каждое множество.
Задача 4. Ок, принято. Правда, множество I ты записал странно. Разумнее было бы записать его сразу в виде I = {зелёный, красный, оранжевый, жёлтый}. Запись, которой ты воспользовался, обычно используется для выделения подмножества из некоторого заданного наперёд множества. То есть пусть есть какое-то множество X. Есть предикат P(x) - сиречь такое утверждение, которое может быть либо истинным, либо ложным в зависимости от x. Мы желаем образовать некое новое множество I, выделив некоторое подмножество из X. Для этого мы пишем I = {x∈X | P(x)}, что означает, что в создаваемое нами множество I будут входить те и только те элементы X, которые обладают свойством P.
Пример.
X - множество целых чисел.
Предикат P означает "чётное". P(2) истинно, P(403) ложно. Тогда множество I, определенное как {x∈X | P(x)}, будет множеством всех чётных чисел.
Предикат Q означает "делится на 8". P(8) истинно, P(403) ложно. Тогда множество I, определенное как {x∈X | Q(x)}, будет множеством всех чисел, делящихся на 8.
Предикаты можно обозначать любым символом, в том числе и многобуквенным, не обязательно использовать только P.
Когда мы будем обсуждать аксиоматику Цермело-Френкеля, мы к этому вернёмся.

Задача 5. Ок, принято. Запись
> ¬(∃a)P((а ∈ A(красный)) ∧ (а ∈ A(зелёный)) ∧ (а ∈ A(оранжевый)) ∧ (а ∈ A(жёлтый))
несколько странная, но это сейчас не принципиально.

Задача 6.
Ок, принято. Дизъюнктивное семейство - это индексированное множество непересекающихся множеств. Поскольку фразу "непересекающиеся множества" можно понимать по-разному, в таких случаях принято говорить "попарно не пересекающихся". Понятно, что если они попарно не пересекаются, то они являются тем, что ты назвал уникальные.

Задача 7.
Я не совсем понял запись. Попрошу выписать это множество явно - то есть записать его элементы через запятую в фигурных скобках, типа {зелёный, красный, оранжевый, жёлтый}.

Задача 8.
Ок, ЕЯПП. Да, конечно, это нормально.

Подумай над задачей 3 и над остальными задачами.

Ещё теория. Множество вида {a,b} называется неупорядоченной парой. Неупорядоченная она потому, что {a,b} = {b,a}.
Заметь, что мы не требуем, чтобы a обязательно отличалось от b. То есть a может быть равно b, тогда {a,b} = {a,a} = {a}. То есть хотя этот объект называется парой, он может быть и из одного элемента.
Имея неупорядоченную пару, можно ввести понятие упорядоченной пары <a,b>. Есть несколько путей конструирования упорядоченной пары, их мы обсудим во время разговора об аксиоматике, пока что я просто скажу, что упорядоченная пара возникает из упорядоченной пары.

Самое интересное свойство упорядоченной пары в том, что она, гм, упорядочена. Если множество {банан, груша} - это то же самое множество, что и {груша, банан}, то упорядоченные пары <банан, груша> и <груша, банан> - это разные пары.
Другое интересное свойство в том, что в упорядоченной паре дубликаты имеют смысл. Пара <a, a> не есть множество {a}.
Упорядоченную пару можно себе представлять как семейство множеств, индексированное множеством {1,2}.
То есть на пару <банан, яблоко> можно смотреть как на список
1. банан
2. яблоко

Так вот. Имея понятие упорядоченной пары, мы можем ввести понятие "декартово произведение множеств".
Декартовым произведением множеств A и B называется множество упорядоченных пар вида <a,b>, где a пробегает множество A, b пробегает множество B. Иными словами, это множество всех возможных упорядоченных пар, в которых на первом месте стоит элемент из A, на втором месте - элемент из B. Декартово произведение A на B обозначается A×B.

Задача 9.
Пусть A = {огурец, арбуз, капуста}, B = {перец, томат}. Явно выписать множество A×B.

Задача 10.
Количество элементов во множестве называется мощностью множества. Если мощность A равна числу x, мощность B равна числу y, то чему равна мощность A×B?
>> No.133870 Reply
>>133869
> упорядоченная пара возникает из неупорядоченной пары.
тоже фикс
>> No.133871 Reply
>>133869
> Моя не понял, так как ты ответишь на задачи 1 и 2? Там либо один и тот же, либо не всегда один и тот же.
Хм, ну, я имел в виду, что два множества - математики и шахматисты с определёнными свойствами вполне могут пересекаться, а могут и не. Мы ведь тут рассматриваем некоторых абстрактных математиков и шахматистов в вакууме, а не занимаемся датамайнингом, сколько из реально существовавших и живущих ныне математиков увлекались шахматами и наоборот? Возможно, я что-то недопонял тогда.
> Правда, множество I ты записал странно.
Да, что-то путаюсь, десу.
>> No.133872 Reply
>>133871
> Мы ведь тут рассматриваем некоторых абстрактных математиков и шахматистов в вакууме
Да, конечно.
> вполне могут пересекаться, а могут и не
Хорошо, что ты подумал о пересечении, это правильное направление. Но не важно, пусто ли это пересечение.
>> No.133873 Reply
>>133872
А по поводу странной записи, если ты об A(i) - просто т.к. индексов не завезли, я подумал, что можно будет записать так, что-то вроде того, как в ЯП записывается сабскрипт массива A[i], но да, похоже, идея не очень удачная. Получилось как-то похоже на функцию... В принципе, я бы мог рендерить записи с хитрыми символами в картинки, если это будет удобно Жалко, что тут нет поддержки латеха, лол, было бы весьма нелишне
>> No.133875 Reply
>>133873
> Получилось как-то похоже на функцию
Так и надо. Индексированное множество - это функция и есть, если работать в ZFC.
>> No.133876 Reply
>>133875
> если работать в ZFC
А что это такое?
>> No.133877 Reply
>>133876
Так называется стандартная аксиоматика теории множеств, Zermelo-Fraenkel with Choice. Дело в том, что оригинальная, канторовская теория множеств оказалась несовершенной. http://rghost.ru/59654216 - можешь посмотреть вот эту книжку, начиная с 12 страницы и примерно до середины второй главы. Глубоко и основательно вчитываться не рекомендую, поскольку этот материал представляет сейчас скорее исторический интерес, чем исследовательский.
Аксиомы ZFC я планирую обстоятельно разобрать с тобой после того, как ты разберёшься с элементарщиной.

ruwiki://Теория_множеств
ruwiki://Кризис_оснований_математики
ruwiki://Парадоксы_теории_множеств
>> No.133879 Reply
File: ladytron-miraaroyo.jpg
Jpg, 6.30 KB, 300×240 - Click the image to expand
edit Find source with google Find source with iqdb
ladytron-miraaroyo.jpg
>>133509
Посоветуете книжку по теории колец?
>> No.133880 Reply
>>133879
Атья, Макдональд, введение в коммутативную алгебру. Начни с этого.
>> No.133881 Reply
File: Снимок34.PNG
Png, 4.60 KB, 176×126 - Click the image to expand
edit Find source with google Find source with iqdb
Снимок34.PNG
Как такие уравнения преобразовывать?
>> No.133882 Reply
>>133881
Положим, что x/92 = a. Знаменатель не может обращаться в нуль, так что a != -10.

a/(10+a) = 0.06

a = 0.06×(10+a)
a = 0.06×10 + 0.06×a
a = 0.6 + 0.06×a
a - 0.06×a = 0.6
a(1 - 0.06) = 0.6
a×0.94 = 0.6
a = 0.6/0.94
>> No.133888 Reply
Аноны, посоветуйте легкую книжку или методичку по исследованию решений уравнений и систем на устойчивость по Ляпунову. Я совершенно не понимаю эту тему. Даже по примерам не могу понять, что там происходит.
>> No.133925 Reply
File: 9aadaee2d686e403a85cf2efaf47adb7.jpg
Jpg, 1041.29 KB, 4250×5290 - Click the image to expand
edit Find source with google Find source with iqdb
9aadaee2d686e403a85cf2efaf47adb7.jpg
y`-2y+y=2xcosx+sinx. Должен же быть способ решить эту баку без дифференцирования дважды многочлена (Ах^2+Bx)cosx+(Cx^2+Dx)sinx. Потом еще всего этого крокодила туда подставлять, не могу без ошибки обойтись. Есть же более короткий путь без маткада?
>> No.133927 Reply
>>133925
А шут его знает. Тебе сильно надо?
>> No.133934 Reply
Первый курс, math major, половину материала не знаю, пинаю хуи, через неделю экзамены. Я вылечу, или все можно заботать за ночь перед экзаменом?
>> No.133935 Reply
>>133934
Вылетишь, скорее всего, только в сентябре. Деканаты не спешат отчислять.
>> No.133936 Reply
>>133934
> math major
Где учишься-то, майор математика?
>> No.133937 Reply
>>133934
Если не долбоеб, то за неделю можно заботать даже небо и Аллаха. Но это нужно собрать яйца в кулак и сесть прям сейчас
>> No.133945 Reply
Да какой-то предел последовательности и предел функции. Надо по определению доказать их. Какой общий принцип таких таких доказательств? Чем отличается первый от второго?
>> No.133946 Reply
File: 7khNqITvlsS81rIWmmJKnw0vF5orJi-origin.jpg
Jpg, 195.18 KB, 700×525 - Click the image to expand
edit Find source with google Find source with iqdb
7khNqITvlsS81rIWmmJKnw0vF5orJi-origin.jpg
>>133945
Последовательность - отображение из натуральных чисел в вещественные.
Функция - отображение из вещественных чисел в вещественные.

Предел последовательности зависит только от одного аргумента - от последовательности.
Предел функции зависит от двух аргументов - от самой функции и от точки, в которой этот предел считается. У одной и той же функции в разных точках разные пределы.
Можно говорить о пределе последовательности, но нельзя говорить о пределе просто функции. Нужно говорить о пределе функции в точке.

Посмотри внимательно на эпсилон-дельта определения. Общий принцип в том, что тебе нужно угадать функцию дельта, зависящую от эпсилон.
>> No.133949 Reply
>>133946
У меня затык на моменте
> угадать
>> No.133950 Reply
>>133949
Щито поделать. Обычно там достаточно как-то повертеть неравенства, использовать теорему о милиционерах и правило Лопиталя.
>> No.133981 Reply
File: Безымянный.png
Png, 10.47 KB, 664×461 - Click the image to expand
edit Find source with google Find source with iqdb
Безымянный.png
>>133509
Как происходят шаги 3-6?
>> No.133985 Reply
>>133981
Ну почти разобрался, но что за хрень происходит на 5-6? Куда пропадает степень?
>> No.133986 Reply
>>133985
> что за хрень происходит на 5-6
x-2 за скобки вынесли.
>> No.133989 Reply
>>133888
Посмотри сначала обыкновенные дифференциальные уравнения Арнольда, чтобы понять смысл всего этого дела, про функцию Ляпунова написано тут enwiki://Lyapunov_stability, ну а потом задачки из Филиппова например.
>> No.133991 Reply
>>133989
> if all solutions of the dynamical system that start out near an equilibrium point xe stay near xe forever, then x_e is Lyapunov stable
Написала какая-то хуйня. Как решения могут оставаться ВСЕГДА возле точки равновесия, если при изменении t изменяется и значения решения? Функция-решение никогда не стоит на месте, она ИЗМЕНЯЕТСЯ.
>> No.133997 Reply
Что такое S(n) в контексте перестановок? Попалась задача: сколько циклов длины k в S(n).
>> No.134002 Reply
>>133997
Количество элементов в множестве.
>> No.134006 Reply
>>133997
Это множество всех перестановок множества из n элементов.
>> No.134018 Reply
>>133613
Есть понятие предел, и его выражают терминами теории. У тебя в надписи — с помощью арифметических и теоретико-множественных понятий. Предел таков, что для любой величины, которую обознача
или эпсилон, с какого-либо номера дальнейшие члены последовательности отличаются от предела меньше, чем на эпсилон.
>>133616 я не понял, вооще, названо многовато необязательных понятий.
>> No.134020 Reply
>>134018
Какие конкретно понятия ты считаешь необязательными?
>> No.134039 Reply
File: lb_i6___visitors_by_hannimble-d67tle4.png
Png, 134.06 KB, 1070×746 - Click the image to expand
edit Find source with google Find source with iqdb
lb_i6___visitors_by_hannimble-d67tle4.png
>> No.134057 Reply
>>133509
Что имеют ввиду, когда пишут f(x+0)?
Пример: Предел f(x) при x->x0+ == f(x+0)
>> No.134058 Reply
>>134057 x плюс бесконечно малое, т.е. к иксу подходим справа. хотя хз, кто так пишет. обычно х+dx или x-dx
>> No.134059 Reply
>>134058
Позволю себе заявить, что вы написали ересь, сударь.
> хз, кто так пишет
Это, как я заметил, очень распространённая запись среди математиков московской школы. Так часто пишут в МГУ и во многих провинциальных вузах России. Хотя я не утверждаю, что только московская школа так пишет.

>>134057
Интервал (p;q) - это, как обычно, множество всех чисел x таких, что p<x<q.

Дельта-окрестностью числа a называется интервал (a-δ; a+δ).
Выколотой дельта-окрестностью числа a называется множество чисел (a-δ; a)∪(a; a+δ).

Левой окрестностью числа x назову интервал (a-δ; a).
Правой окрестностью числа x назову интервал (a; a+δ).
Таким образом, выколотая дельта-окрестность числа a - это просто объединение левой окрестности и правой окрестности.

Пусть есть числовая функция f, и пусть есть какое-то множество чисел M. Образом множества M мы будем называть множество всех чисел f(x), где x принадлежит множеству M. Обозначается как f(M).

Как известно, число A называется пределом функции f в точке a, если в любой окрестности точки A лежит образ хотя бы одной выколотой окрестности точки a. Это обозначается как lim(x->a)f(x) = A.

Число A называется пределом справа функции f в точке a, если в любой окрестности точки A лежит образ хотя бы одной правой окрестности точки a. Это обозначается как lim(x->a+0)f(x) = A.

Число A называется пределом слева функции f в точке a, если в любой окрестности точки A лежит образ хотя бы одной левой окрестности точки a. Это обозначается как lim(x->a-0)f(x) = A.

f(x+0) есть предел справа функции f в точке x. Символически, f(x+0) := lim(x->a+0)f(x).
f(x-0), аналогично, есть предел слева.

f(x+0)=b в эпсилон-дельта нотации запишется, например, как
∀ε>0 ∃δ>0 ∀x (x∈(a; a+δ)) ⇒ (|f(x)-b|<ε))

Иногда левым называют правое, а правым - левое, поэтому всегда нужно уточнять, о чём идёт речь.
Как обычно, извиняюсь за возможные ляпы.
ruwiki://Односторонний_предел
>> No.134060 Reply
>>134059
О, скобка лишняя. f(x+0)=b есть
∀ε>0 ∃δ>0 ∀x (x∈(a; a+δ) ⇒ (|f(x)-b|<ε))   Так правильнее.
>> No.134062 Reply
>>134059
> ересь
вы тоже самое написали, только словами из учебника
> вузах России
ну, с наукой здесь всё плохо вот уже как 40 лет. ничего удивительного, сударь
позвольте узнать, когда же ноль перестал быть конкретной точкой в числовом пространстве, что запись x+0 стала тождественна x+dx?
>> No.134064 Reply
>>134062
Нет, я написал не то же самое. Объясни, что ты понимаешь под dx и под бесконечно-малым же. Ты написал не ересь только если ты использовал эти термины в каком-то отличном от общепринятого смысле.
>> No.134066 Reply
File: я все понял.jpg
Jpg, 164.85 KB, 400×359 - Click the image to expand
edit Find source with google Find source with iqdb
я все понял.jpg
>>134059
Спасибо, няша.
>> No.134077 Reply
>>134059
> очень распространённая запись среди математиков московской школы.
Вспомнилось, как препод по матану использовал термины "отображение на", "отображение в", "точка сгущения", хотя напрашивались все-таки все любимые иностранные аналоги.
>> No.134079 Reply
>>134077
А что такое точка сгущения, кстати? Это предельная точка или точка конденсации?
>> No.134082 Reply
>>134079
Предельная. Вообще это все из-за того, что основы общей топологии впихивают в курс классического анализа, причем на околоинтуитивном уровне. А преподы старой школы вообще ничего не знают, будто и нет такой науки, как общая топология.
>> No.134090 Reply
File: кольца.-поля.png
Png, 42.55 KB, 744×358 - Click the image to expand
edit Find source with google Find source with iqdb
кольца.-поля.png
>>134082
Они знают о существовании всего этого, они просто считают, что это всё ненужная ерунда, а нужно брать интегралы и решать дифуры.
>> No.134091 Reply
>>134090
> пик
И эту вашу топологию фарисейскую запретить!
>> No.134092 Reply
>>134090
>>134091
Ну, положим, для прикладных задач все эти доказательства и не нужны, нужны только методы и готовые знания, принимаемые за аксиомы.
мимопроходил
>> No.134093 Reply
>>134092
> лекало, таблицы квадратных уравнений, да вера в Боженьку - что ещё для развития нанотехнологий нужно?
>> No.134094 Reply
>>134092
но прикладные задачи может посчитать и калькулятор, зачем в них требуется человек?
>> No.134098 Reply
File: 1245943713902.png
Png, 0.12 KB, 1×1 - Click the image to expand
edit Find source with google Find source with iqdb
1245943713902.png
>>134090
Продвинутые матфизики бы заплевали автора пика с ног до головы.
>>134094
Не зачем. Не нужен. А вот прикладные задачи человеку нужны. Для лучшего понимания теоритического материала.
>> No.134105 Reply
>>133509
Анон, а как решать уравнения вида a*x = b (mod z) ?
>> No.134107 Reply
>>134098
> Продвинутые матфизики бы заплевали автора пика с ног до головы
Матфизика и физика - разные вещи. Физики бы тоже заплевали, впрочем.
мимофизик
>> No.134109 Reply
>>134105
Перепишем в виде ax = b + nz, n целое
Если НОД(a, z) = g, b не делится на g, то решений нет
Иначе сокращаем на g: (a/g)x = (b/g) + n(z/g)
Переобозначим A = a/g, B = b/g, Z = z/g: Ax = B + nZ, n целое
Это то же самое, что решить A*x = B (mod Z), но теперь НОД(A, Z) = 1
Используя алгоритм Евклида, находим X, Y: AX + ZY = 1
x = BX - искомое
>> No.134110 Reply
>>134105
>>134109
Нет, соврал немного:
решение не только x = BX, но ещё BX+Z, BX+2Z, ... BX+(g-1)Z
>> No.134113 Reply
>>134109
Еще вопрос, как решать такие:
a^n=b(mod x)
>> No.134114 Reply
>>134113
Нужно найти x?
Перепишем в виде a^n-b=0 (mod x)
Это то же самое, что a^n-b делится на x, т.е. x - произвольный делитель a^n-b
>> No.134128 Reply
помогите ребятки,объясните как решать y-xy'=1+x^2*y
>> No.134130 Reply
Вот есть уравнение плоскости( в координатном методе), а есть ли уравнение окружности, чтоб для вектора нормали.
>> No.134134 Reply
File: Untitled.png
Png, 7.82 KB, 654×98 - Click the image to expand
edit Find source with google Find source with iqdb
Untitled.png
Как?...
>> No.134143 Reply
>>134134
Убрали из суммы первые два члена. Импликация куда интереснее.
>> No.134147 Reply
File: Untitled.png
Png, 60.79 KB, 814×373 - Click the image to expand
edit Find source with google Find source with iqdb
Untitled.png
>>134143
Кхм, пардон. Стоило сразу всё задание показать. До меня так и не дашло, почему два выражения равны.
>> No.134148 Reply
>>134147
Ну, там получается cosx =1- f(x)*x^2, откуда и получаем равенство.
>> No.134149 Reply
>>134147
Там же х в степени 2n +2, a не 2n
>> No.134154 Reply
Аноны, где применяются несобственные интегралы и (несобственные) интегралы, зависящие от параметра? Зачем это нужно?
>> No.134165 Reply
>>134154
Преобразование фурье, например. Используется в радиофизике, оптике и еще дофига где.
>> No.134178 Reply
File: pic.JPG
Jpg, 4.05 KB, 292×91 - Click the image to expand
edit Find source with google Find source with iqdb
pic.JPG
Анон, где можно ознакомится с правилами дифференцирования линейных объектов на подобии пикрелейтида?

Понятно, что можно руками высчитать матрицу дифференциалов покомпонентно, а потом найти их линейное представление. Но мне не кажется, что все этим занимаются, явно есть какая-то методика решение, может через свёртки или что-то ещё?
>> No.134179 Reply
Прошу прощения, я конечно не по адресу, но может вы подскажите. А есть нефтегазовая кафедра?
>> No.134181 Reply
>>134179
> нефтегазовая кафедра
Вам в политикотред в /b/
>> No.134191 Reply
>>134148
Вот меня смущает, что получается. Я не понимаю почему левая сторона равна правой. Единица какая-то, всё это...
>> No.134192 Reply
>>134181
Я между прочим серьёзно. Знали бы вы как сложно найти качественную литературу, не говоря уже о материалах для диплома.
>> No.134200 Reply
>>134191
Выносим из суммы члены, которые в сумме равны единице. Все.
>> No.134210 Reply
File: ss+(2014-12-23+at...
Png, 28.99 KB, 1436×300
edit Find source with google Find source with iqdb
ss+(2014-12-23+at+09.45.38).png
File: Date_A_Live_Poste...
Png, 7967.63 KB, 2500×1696
edit Find source with google Find source with iqdb
Date_A_Live_Poster_Image.png

Уважаемые математики, мне нужно сделать лабу, а я туплю и не могу рассчитать. Помогите, пожалуйста. Постараюсь выразиться как можно проще и доступнее.

Рассчитать количество жизнеспособных бактерий в 1 мл. суспензии каждого варианта и выразить выживаемость в % от контроля. Данные внести в таблицу, вычертить кривые выживаемости в зависимости от дозы облучения в % от контроля.
>> No.134220 Reply
>>134210
конечное разведение чего? что это вообще за параметр? сколько мл суспензии было в каждом опыте? сколько бактерий в каждой группе?
>> No.134223 Reply
>>134220
> конечное разведение чего?
Бактерий в "воде".
> сколько мл суспензии было в каждом опыте?
См. табл. "разведение в воде".
> сколько бактерий в каждой группе?
Итого см. табл. "количество выросших колоний".
>> No.134232 Reply
>>134223
Если так будет понятнее: взяли бактерии с водичкой (разной концентрации), их разлили по разным чашкам Петри (для каждой концентрации - своя чашка). Ну и потом подсчитали количество выросших колоний на чашках Петри.
>> No.134251 Reply
>>134232
Что такое конечное разведение? Почему оно выражено в числах, что означают эти числа?
>> No.134253 Reply
Ах, Доброанон, у меня сегодня модуль по матану, потом зачёт. Скажи, как же ты всё это понимаешь? Есть ли какой-то секрет? Просто зубрёж? И неужели мне, таки, место в биореакторе?
>> No.134254 Reply
>>134253
> матан
> зубрёж
Нет пути. Не можешь уложить в голове пару банальных принципов штолей?
>> No.134256 Reply
>>134254
Пытаюсь, но часто ошибаюсь по невнимательности. Кажется, это не моё. Хорошо, что только один семестр остался, иначе, чувствую, лететь мне из универа на вольные хлеба!
>> No.134257 Reply
>>134178
А что тебе нужно-то? Хочешь теорию - Лоран Шварц, в основном первый том. Хочешь сделать калькулятор - почитай что-нибудь о численных методах.

>>134179
Здесь, в /u/, был тред, но в нём отписывались полтора анона, и он даже в путеводитель не попал.

>>134253
На ментальном уровне математика выглядит так же, как программирование. Из программирования известны такие концепции как класс, объект, метод, пространство имён, функция. Всё это - частные случаи математических объектов. Отличие математики от программирования чисто количественное. В программировании используется от силы пара десятков концепций, о которых известно сравнительно мало фактов. В самом деле, что интересного можно сказать об объекте? Чтобы стать успешнее в математике, тебе нужно научиться без проблем работать с сотнями концепций. Тренируйся - научишься. Абстрактное мышление развивается медленно, но развивается же. Если ты умеешь программировать хотя бы чуть-чуть, то у тебя есть абстрактное мышление.

>>134256
Школы становятся всё хуже и хуже, похоже.

мимо прополз
>> No.134262 Reply
File: nGoZVVmgCEA.jpg
Jpg, 48.77 KB, 613×440 - Click the image to expand
edit Find source with google Find source with iqdb
nGoZVVmgCEA.jpg
Аноны, направьте меня на путь истинный?
>> No.134263 Reply
>>134262
Поклянись, что прочитаешь учебник от корки до корки.
>> No.134270 Reply
>>134262
Проще этих заданий ничего нет. Тебе должно быть стыдно.
>> No.134275 Reply
>>133869
> Задача 7.
> Я не совсем понял запись. Попрошу выписать это множество явно - то есть записать его элементы через запятую в фигурных скобках, типа {зелёный, красный, оранжевый, жёлтый}.
а = {1, 2, 3, 4, 5, 6, 7, 8, 9, 10};
У меня там получился "погроммизм головного мозга", обозначение диапазона через (х > m) И (x < n). Правильнее было бы наверно {a ∈ A | 1 ⩽ a ⩽ 10}
> Моя не понял, так как ты ответишь на задачи 1 и 2? Там либо один и тот же, либо не всегда один и тот же.
Так, мкау. Попробую как-то более формализовать ответ что ли. Пусть есть множество А - математики и множество В - шахматисты, такие, что A ⊂ M и B ⊂ M, множество X = {x | x ∈ A, B} - это множество всех шахматистов-математиков. Очевидно, что X = A ∩ B, пусть элементы множества обладают свойством Р - возраст. Раз мы оперируем таким понятием, как старейший, придётся допустить, что есть такое максимальное Pmax, что ни одно P ≱ Pmax, тогда множество старейших шахматистов-математиков Z = {z ∈ X | P(z)=Pmax}. Pmax у нас уникально, элемент будет только один и этот элемент z ∈ A, B. Значит старейший хашматист среди математиков и старейший математик среди шахматистов - это один и тот же человек.
Заменяя свойство "возраст" на свойство "успешность" или "лучшесть" - не знаю, как его обозвать, формально получим тот же ответ для задачи 2. Но тут есть момент, который несколько смущают, как мне кажется. Можно быть лучшим шахматистом и заниматься математикой, а можно быть лучшим математиком и играть в шахматы и тогда, вообще говоря, нам стоило бы различать два свойства - успехи в математике и успехи в шахматах. На этом основании наверно было бы уместнее также разделить людей по их основному занятию. Кто-то хорош в шахматах и интересуется математикой, но не является отменным математиком и наоборот, кто-то является хорошим математиком и увлекается шахматами, но играет посредственно. Так что решение получается довольно формальным. Насчёт возраста: тут пришлось исходить из того, что возраст - это некое уникальное значение. И действительно, наверно трудно найти двух и более людей, родившихся в одно и то же время с точностью до миллисекунды, допустим (сама постановка вопроса о такой точности уже немного абсурдна). Обычдно мы отсчитываем возраст в годах и множество старейших шахматистов-математиков будет состоять из более, чем 1 элементов, так что в целом было бы уместнее говорить о старейших шахматистах-математиках в общем случае. Я понимаю, что это всё не очень существенно в задачах, но просто сбивает с толку что ли... Поэтому я сначала написал так, как написал.
> Задача 3. Подумай ещё. Количество элементов во множестве - мощность. Мощность есть у каждого множества, а вот измеримым является-таки не каждое множество.
Мы можем говорить о каком-то неравенстве только если по крайней мере |A|, |B| > 12. В противном случае, либо шахматистов (В) окажется больше, либо метаметиков и щахматистов будет поровну (1). Если |A| >> |B|, очевидно, что математиков (А) окажется больше. В идеале, если |A| = |B| и оба множества неисчислимые(?), то больше будет шахматистов (В), но это уже абсурд, потому что множество людей в целом исчислимо, и шахматистов/математиков ну никак не может быть больше, чем всех людей. В общем, сдаюсь. По-моему, чтобы дать однозначный ответ не хватает данных. Наверно я опять чего-то не понимаю sigh
>> No.134276 Reply
>>134275
Задачи 7 и 1 - ок.
В задаче 2 ты правильно сомневаешься. На пересечении множества математиков и множества шахматистов заданы два, вообще говоря, разных порядка. Мы упорядочиваем это пересечение отдельно по успешности в шахматах, отдельно по успешности в математике. Поэтому "лучшие" люди могут быть разными.
Задача 3. Обозначь количество элементов во множестве математиков как x, количество элементов во множестве шахматистов как y, количество элементов в пересечении множества математиков и множества шахматистов как z. Затем вырази z через x, вырази z через y.
Можешь считать, что, например, если каждый десятый ондатр - выхухоль, то ондатров ровно в десять раз больше, чем выхухолей.

Декартово произведение понятно?
>> No.134277 Reply
>>134276
Честно говоря, я пока не брался за остальную часть, просто хотел хотя бы по этому поводу отписаться. А то ведь и так пропадаю на многие дни. В общем, чтобы не создавалось впечатление, что я совсем уж с концами исчез. Теперь буду смотреть произведение и там ещё из старого остались кой-какие хвосты.
>> No.134278 Reply
File: _009____123___naptime_by_akityu-d58ziou.png
Png, 119.86 KB, 985×812 - Click the image to expand
edit Find source with google Find source with iqdb
_009____123___naptime_by_akityu-d58ziou.png
>>134277
Оки.
>> No.134281 Reply
File: pic.jpg
Jpg, 76.53 KB, 746×694 - Click the image to expand
edit Find source with google Find source with iqdb
pic.jpg
>>134257
> А что тебе нужно-то?
Задачи, скорее, прикладного характера. Просто ищу способ упростить себе жизнь. Нужно рассчитать несколько МНК, вот только конструкции там порой довольно громоздкие, не всегда уверен в правильности получающихся дифференциалов. Приходиться проверять себя на каких-то простых примерах, проделывая пикрелейтид и.т.п. Не смертельно, но времени тратится довольно много.
>> No.134286 Reply
http://www.topuniversities.com/university-rankings/university-subject-[...]arch=
Такой вот вопрос: если идет выбор между мехматом и одним из вузов на 10-20 строчек ниже его, то что лучше выбрать? В европы, как я слышал, модульная система.
>> No.134301 Reply
Здравствуйте умные доброаноны. Так получилось что я заканчиваю девятый класс и абсолютно не знаю математику. Сначала было лень заниматься ей по своей глупости. Теперь я наконец то осознал что это очень важно. С 5 класса можно считать, что я не ходил на матем. Не знаю таблицы умножения и всяких элементарных вещей типа дробей и уравнений. Что мне делать в моей ситуации? Ведь скоро экзамен. Да и самому неприятно от того что не знаю предмет. Как мне дальше то учиться во всяких колледжах и вузиках. К репетитору идти и стыдно и страшно, так как я замкнутый и тесный контакт с человеком для меня тяжел.
>> No.134327 Reply
>>134301
Выучить таблицу умножения, прочитать учебники.
http://www.alleng.ru/edu/math1.htm

Я могу поговорить с тобой обстоятельнее. Если хочешь, то оставь здесь джаббер или аську.
>> No.134328 Reply
>>134301
«Алгебра» Гельфанда и Шеня. Правда, там всё чуть ближе к настоящей математике, чем школьной, и соответственно сложней, но хотя бы попробовать стоит. И в ней нет некоторых тем, вроде. Логарифмы и тригонометрию придется учить по другим ресурсам. «Математике абитуриенту» Ткачука, например. Планиметрию можно учить по книге Прасолова «Задачи по планиметрии». Если не ошибаюсь, она начинается с самых-самых азов, таких как эквивалентности треугольников. Если знаешь инглиш, то тебе однозначно сюда: https://www.khanacademy.org/

Ну, и у меня самого есть вопросик: как правильно учить алгебру? Начала теории групп. Нужно зубрить определения, тренироваться их доказывать, доказывать эквивалентность разных? Или как вообще должен выглядеть алгоритм для запоминания и понимания.
>> No.134329 Reply
>>134328
Если начала, то там определения общепринятые, и их, таки да, нужно зубрить. Доказывать эквивалентность - само собой. Нужно ещё доказывать всякие простые факты, например, что если в кольце ax=ay и a не является делителем нуля, то на a можно сократить. Листочки из http://dobrochan.com/src/rar/1410/Тривиум.rar порешать, задачи из Проскурякова посчитать, всё такое.

Вообще, ты сначала должен поставить перед собой конкретную цель, а потом для каждого совершенного тобой действия решать, приблизился ли ты к этой цели или нет. Совершать надо те действия, которые приближают тебя к цели. Если действия не приближают тебя к цели, то такие действия не надо совершать. Чем конкретнее цель, тем лучше. Цель "выучить начала теории групп" никуда не годится например, потому что слишком размыта.
>> No.134331 Reply
>>134327
Аська 669839382
>> No.134334 Reply
>>134329
Хм. Спасибо за ответ, анон. Про цель ты очень точно сказал. Именно этого мне очень сильно не хватает. Я плохо понимаю зачем именно я учу то, что учу. Такая же проблема с аналгемом: я очень сильно сомневаюсь, что десятки примитивных формул помогут лучше понимать потом топологию, механику я буду учить не скоро, а зачем еще он нужен — понятия не имею. С матаном всё более очевидно, поэтому и переваривается он гораздо проще.
>> No.134338 Reply
>>134278
Поспешу отписаться по остатку, благо там, кажется, ничего затруднений не вызвало:

(>>133869)
> Задача 9.
> Пусть A = {огурец, арбуз, капуста}, B = {перец, томат}. Явно выписать множество A×B.
A×B = {<огурец, перец>, <огурец, томат>, <арбуз, перец>, <арбуз, томат>, <капуста, перец>, <капуста, томат>};
> Задача 10.
> Количество элементов во множестве называется мощностью множества. Если мощность A равна числу x, мощность B равна числу y, то чему равна мощность A×B?
|A×B| = xy; Или вообще говоря, |A×B| = |A|∙|B|; По определению у нас получается A×B = {<a,b> | a ∈ A ∧ b ∈ B}, каждый элемент a ∈ A образует упорядоченную пару с каждым элементом b ∈ B, а значит, общее число упорядоченных пар это произведение мощностей множеств A и B.
>> No.134346 Reply
File: вершины.png
Png, 6.94 KB, 647×322 - Click the image to expand
edit Find source with google Find source with iqdb
вершины.png
>>134338
Отлично.
Подобно декартову произведению двух множеств, можно ввести декартово произведение трёх, четырёх, ... , n множеств. Их элементами будут соответственно упорядоченные двойки, упорядоченные тройки, ... , упорядоченные n-ки. На само множество можно взглянуть как на декартово произведение одного множества. В программировании упорядоченная n-ка обычно называется массивом из n элементов.

Пусть есть какие-то множества A и B. Подмножества их декартова произведения называются отношениями <A,B>, или отношениями вида A,B, или, точнее, бинарными отношениями <A,B>. Бинарное отношение - это просто какое-то множество пар, таким образом. Отношение может быть пустым, отношение может совпадать с A×B.

Пусть некоторое отношение обозначено буквой φ, то есть φ ⊂ A×B. Если пара <a,b> принадлежит отношению φ, то мы будем писать a φ b. То есть <a,b> ∈ φ ⇔ a φ b. Читается как "a и b находятся в отношении φ". Иногда будем пользоваться записью φ(a,b).

Пример 1. Рассмотрим множество целых чисел ℤ. Рассмотрим его "декартов квадрат" ℤ×ℤ. В курсе теории чисел описывается некоторое подмножество этого декартова квадрата, обозначаемое как <. Это - известное со школы отношение "меньше". Оно, конечно, описывается не перечислением элементов, потому что целых чисел бесконечно много. Оно описывается с опорой на аксиомы. Об этом описании и вообще о том, что такое числа, мы поговорим в будущем. Утверждение, что 5 < 7, является утверждением, что <5, 7> ∈ <.

Пример 2. Рассмотрим множество, которое ты построил в задаче 9, и рассмотрим его подмножество t = {<огурец, перец>, <арбуз, перец>, <арбуз, томат>}. Это вот тоже отношение. Верно, что огурец t перец. Неверно, что перец t огурец, ведь пара <перец, огурец> не принадлежит t.

Отношения, определённые на декартовом произведении одного множества (то есть просто его подмножества) называются унарными отношениями.
Отношения, определённые на декартовом произведении двух множеств, называются бинарными отношениями.
Отношения, определённые на декартовом произведении трёх множеств, называются тернарными отношениями.

Задача 11. Придумай какое-нибудь небольшое тернарное отношение и явно выпиши все его элементы в виде φ(a,b,c).

Далее я хочу ввести важное понятие отношения эквивалентности на M. Пусть φ обозначает какое-то бинарное отношение вида <M, M>.
Отношение φ называется отношением эквивалентности, если оно обладает тремя свойствами.
1. aφa, так называемая рефлексивность. Любой элемент из M находится в отношении φ сам с собой.
2. Если a φ b, то b φ a. Так называемая симметричность.
3. Если a φ b и b φ c, то a φ c. Так называемая транзитивность.
Отношение эквивалентности - это небольшое обобщение отношения равенства. Само отношение = тоже является отношением эквивалентности. Действительно, a=a, a=b→b=a, (a=b и b=c) → a=c для любых a,b,c.
Другим примером отношения эквивалентности является "равенство" фигур в школьной геометрии.

В языке программирования PHP есть отношение ==. Оно не является отношением эквивалентности, потому что в нём не выполняется транзитивность. Например, "0" == 0, 0 == "", но неверно, что "0" == "".

Понятие отношения очень важно для теории баз данных. Информатики придумали специальные названия для многих разных отношений, выстроили сложную терминологию с кучей смысловых оттенков. Жаль только, что теорем у информатиков меньше, чем терминов.

Задача 12. Придумай какое-нибудь отношение эквивалентности на некотором угодном тебе множестве.

Теорема. Всякое отношение эквивалентности задаёт некоторое разбиение множества M на "классы эквивалентности".
Вот иллюстрация к теореме безумный навык рисовательства, ага. Две точки эквивалентны, если есть путь из первой точки во вторую. Петельки нужны для рефлексивности. Видно, что на картинке три класса эквивалентности.

Теперь доказательство.
1. Пусть φ - отношение эквивалентности на M. Каждому элементу множества M сопоставим множество всех тех элементов, с которыми он находится в отношении φ. Получим покрытие множества M. Это действительно покрытие, поскольку каждый элемент из M находится в отношении φ сам с собой и поэтому покрывается хотя бы одним множеством из построенного семейства. Обозначим это покрытие как N. Любопытно, что множеством индексов этого покрытия является само множество M.

2. В семействе общего вида, как я говорил, допустимы дубликаты. Например, может быть так, что одно и то же множество X входит в семейство и с индексом i, и с индексом j, i≠j. Исключим такую ситуацию, запретив в N дубликаты, то есть если множество входит в семейство N более чем с одним индексом, то мы оставим только одно вхождение, а все остальные выкинем. От этого N не перестанет быть покрытием. Поскольку в семействе теперь нет дубликатов, мы можем утверждать, что если множества равны, то их индексы совпадают. Если индексы не равны, то не равны и множества.

Теперь я вот утверждаю, что N будет разбиением. Докажем это.

3. По нашему определению, разбиением называется такое семейство A с индексами I, что если i ≠ j, то Ai ∩ Aj = ∅. Нам требуется показать, что если два множества из N не равны (это в силу сказанного выше эквивалентно неравенству индексов), то эти множества не пересекаются. Или, что то же самое, что если два множества пересекаются, то они равны.

4. Пусть A и B - два множества из N, пусть их пересечение не пусто.
Присмотримся к пересечению повнимательнее. В нём есть по крайней мере один элемент, x ∈ A⋂B. Этот икс принадлежит как A, так и B. По построению, все элементы A находятся в отношении φ с иксом, равно как и все элементы B.
Пусть a - произвольный элемент A. Тогда a φ x. Пусть b - какой-то элемент из B, тогда x φ b. Тогда a φ x и x φ b - следовательно, по транзитивности a φ b. Отсюда a принадлежит множеству B. Таким образом, A ⊂ B.
Аналогично, B⊂A.
Так как A⊂B и B⊂A, по определению равенства множеств A = B. То есть если A и B пересекаются, то они совпадают, что и доказывает, что N - разбиение.

Если ты не видел раньше эту теорему, то через некоторое время воспроизведи доказательство самостоятельно на листочке (берём множество с отношением эквивалентности, 1 - вводим покрытие, 2 - выкидываем дубликаты, 3 - ссылаемся на определение разбиения, 4 - используем транзитивность). Я написал слишком слов для такого простого факта, это меня смущает. Даже не прочитал нормально, что написал, тороплюсь сейчас. Если есть вопросы, обязательно спрашивай. В этом доказательстве самое интересное - шаг 4, использование транзитивности. Транзитивность встречается много где, так что доказательства вроде этого попадаются довольно часто.

Можно доказать и в некотором смысле обратную теорему: всякое разбиение задаёт отношение эквивалентности. Элементы a и b объявляются эквивалентными, если принадлежат одному и тому же множеству из разбиения.

Задача 13. Выпиши все классы эквивалентности отношения из задачи 12.

Множество всех классов эквивалентности множества M называется фактор-множеством M по отношению φ, или, реже, группировкой M по φ. Можно представлять себе фактор-множество M как само M, в котором эквивалентные элементы не различаются. Фактор-множество множества с моей картинки - это множество из трёх элементов. Фактор-множества широко используются в математике для построения всяких фактор-структур - фактор-групп, фактор-колец, etc. Фактор-множества интересны ещё и с философской точки зрения, поскольку дают основание считать эквивалентные объекты одним и тем же объектом. Фактор-множество - это богатая идея, хотя и простая.

Задача 14. Перепиши фактор-множество из задачи 13, используя в качестве символа множества какой-нибудь его элемент. Например, {1,2,3} обозначь символом 1.
>> No.134354 Reply
File: Гротендик.jpg
Jpg, 134.33 KB, 578×665 - Click the image to expand
edit Find source with google Find source with iqdb
Гротендик.jpg
Все решаете, матаны?
>> No.134375 Reply
>>134328
Тригонометрию и логарифмы можно учить по соответствующим брошюрам МНМЦО, вроде даже за авторством Шеня.
>> No.134397 Reply
File: zzz.png
Png, 17.67 KB, 576×104 - Click the image to expand
edit Find source with google Find source with iqdb
zzz.png
>>133509
Правда ли, что группа ( H,+,e ) является подгруппой группы ( G,+,e ), если H подмножество G?
>> No.134398 Reply
>>134328
Такой тогда вопрос еще. Касается не столько математики, сколько всех точных наук в целом (химия, физика).
Как лучше учить? Банально следовать методу "как в школе", то есть прочитывать параграф, запоминать главное и все формулы, решать большую часть задач и переходить к следующему? Или есть менее болезненный способ?
Просто я гуманитарий с более пространственным мышлением, поэтому просто втиснуть в голову формулы для меня трудновато, а в большинстве школьных учебников занимаются именно втискиванием, а не объяснением того, что от чего пошло и как работает.
Тоже сижу сейчас и понимаю, что ни черта не знаю физику (разве что теорию). В химии тоже только общую теорию и примерно как коэффициенты расставлять. Математику еще более менее понимаю, потому что трудами невыносимыми вдолбил в голову уравнения, плюс, функции понимать проще.
мимострадающий
>> No.134399 Reply
>>134397
Мы можем взять в аддитивной группе целых чисел множество {6,7} и сделать его группой, объявив 6+6=6, 6+7=7+6=7, 7+7=6. Однако {6,7} не подгруппа группы целых чисел.
Недостаточно, чтобы подмножество просто было группой. Нужно, чтобы оно было группой относительно тех же операций. Если твоя запись это и означает, то ты прав.
>> No.134400 Reply
>>134399
Означает, спасибо.
>> No.134412 Reply
File: choco_donut_by_peonypink-d7u5f39.png
Png, 264.77 KB, 441×600 - Click the image to expand
edit Find source with google Find source with iqdb
choco_donut_by_peonypink-d7u5f39.png
>>134346
Анон, так что, вопросы есть?
>> No.134430 Reply
File: mindfuck.png
Png, 27.27 KB, 614×235 - Click the image to expand
edit Find source with google Find source with iqdb
mindfuck.png
Анон, выручай. Пожалуйста помоги по пункту 2. У меня скоро нервный тик от этой хуйни начнётся. Пусть a/b > a + c/b + d > c/d, a + c/b + d = x/y, тогда ay - bx = 1, -cy + dx = 1. А дальше-то что?
>> No.134432 Reply
>>134412
Да, наверно есть.

(>>134346)
> Задача 11. Придумай какое-нибудь небольшое тернарное отношение и явно выпиши все его элементы в виде φ(a,b,c).
Если здесь, кажись, всё достаточно невинно:
A = {a,b}, B = {u,v}, C = {x,y};
A×B×C = {<a,u,x>, <a,u,y>, <a,v,x>, <a,v,y>, <b,u,x>, <b,u,y>, <b,v,x>, <b,v,y>};
φ(a,u,x), φ(a,u,y), φ(a,v,x), φ(a,v,y), φ(b,u,x), φ(b,u,y), φ(b,v,x), φ(b,v,y);

То вот тут
> Задача 12. Придумай какое-нибудь отношение эквивалентности на некотором угодном тебе множестве.
я сломался. Выше написано, что отношение - это подмножество декартова произведения. мкау. Т.е., это просто упорядоченная пара из кучи пар (для бинарных отношений), правильно? Если я возьму какое-то множество A = {a, b}, сделаю вот так
A×A = {<a,a>, <a,b>, <b,a>, <b,b>};
a, b ∈ A состоят в каких-то отношениях a φ a, a φ b, b φ a, b φ b. В принципе, ничего больше об отношении нам неизвестно. Как на этом абстрактном в вакууме понятии сделать что-то транзитивное или симметричное? Симметричность означает, что наряду с парой <a,b> ∈ A×A есть пара <b,a> ∈ A×A? Тогда транзитивное отношение потребует <a,b>, <b,c>, <a,c> ∈ A×A? Я как-то не очень понимат ;_; Не мог бы ты показать какой-то конкретный пример?
>> No.134446 Reply
>>134430
Пусть a/b < c/d. То есть ad < cb (это определение неравенства дробей, которое полезно запомнить).
Сначала докажем, что a/b < (a+c)/(b+d).
Нужно доказать, что a(b+d) < (a+c)b.
Раскроем скобки в неравенстве, которое нужно доказать. ab+ad < ab+cb.
Вычтем слева и справа ab; неравенство от этого не изменится. Но ad < cb по условию.
Теперь докажем, что (a+c)/(b+d) < с/b ...

Пусть а/b и c/d соседние. Тогда ad - bc = ±1.
Докажем, что a/b и (a+c)/(b+d) соседние.
Для этого требуется доказать, что a(b+d) - b(a+c) = ±1.
Просто раскроем скобки.
ab + ad - ab - bc = ad - bc = ±1.

>>134432
> Задача 11
Хорошо, правильно.
> я сломался.
Просто веру в себя потерял. Ты боишься, а ты не бойся. Ты в силах это понять.
> Т.е., это просто упорядоченная пара
Это какое-то множество упорядоченных пар, не обязательно одноэлементное. А декартово произведение - множество всех возможных упорядоченных пар.
> сделаю вот так
То получишь отношение эквивалентности.
> Я как-то не очень понимат
Возможно, тебе кажется, что в A должно быть три элемента, а их только два. Ты не понимаешь, где взять c, вот ты и смущён. Но в транзитивности ведь не сказано, что a, b и c обязаны различаться. Сказано, что какими бы ни были a, b, c, свойство транзитивности должно выполниться. На место этих букв вполне можно подставить одну и ту же букву же. Если во множестве только два элемента, то среди трёх букв a,b,c, обозначающих элементы этого множества, по крайней мере две будут обозначать один и тот же объект.
Чтобы было нагляднее, переобозначь элементы в A как {x,y}. Тогда станет проще понять, что a,b,c - это чисто технические буквы, формальные.
> Как на этом абстрактном в вакууме понятии сделать что-то транзитивное или симметричное?
Очень просто. Добавил в отношение пару <a,b> - добавь ещё и пары <a,a>, <b,a> и <b,b>.
Добавил в отношение пару <b,c> - добавь сверх того ещё и пару <a,c>.
> конкретный пример
{
<a,b>,
<b,c>,

<a,a>,
<b,a>,
<b,b>,

<c,b>,
<c,c>,

<a,c>,
<c,a>
}

Кстати, эквивалентность двух элементов традиционно обозначают символом ~. a~b, b~c, a~c.

Теперь сделай отношение эквивалентности, в котором по меньшей мере два класса. Хинт: можно построить точки на картинке и эквивалентные точки соединить линиями, а потом просто описать полученное.
> абстрактном в вакууме
Это вовсе и не абстрактное, тут конкретные буковки есть же, которые даже можно явно выписать.
>> No.134447 Reply
>>134446
> То получишь отношение эквивалентности.
Кстати, на одном и том же множестве можно придумать, вообще говоря, несколько отношений эквивалентности. Декартово произведение всегда будет отношением эквивалентности, потому что оно содержит вообще все возможные пары. Но это не очень интересный случай, интересные отношения эквивалентности обычно содержат только часть возможных пар.
>> No.134484 Reply
>>134446
Спасибо, анончик!
>> No.134502 Reply
Математики, я все хочу знать на тему "смешных" дробей. 16/64 = 1/4, 19/95 = 1/5, 26/65 = 2/5. Работает, ведь. Начнем вот, к примеру, с сокращения одной цифры. Тривиальных дробей вида xx...x/xx...x - бесконечно, но если в дроби есть разные цифры? Как оно распределено? А если потом начать сокращать цифры из произвольных мест числителя и знаменателя? Вопросов очень много получается. Все это очень естественно, публикации наверняка есть, но вот беда: я не знаю как их искать.
>> No.134519 Reply
File: 4455578_283790502.jpg
Jpg, 52.94 KB, 500×703 - Click the image to expand
edit Find source with google Find source with iqdb
4455578_283790502.jpg
>>134446
> Возможно, тебе кажется, что в A должно быть три элемента, а их только два. Ты не понимаешь, где взять c, вот ты и смущён.
Именно так всё и было.
> Теперь сделай отношение эквивалентности, в котором по меньшей мере два класса.
Некоторое X = {<a,b>, <b,c>, <c,a>, <d,e>, <e,f>, <f,d>};
> Задача 13. Выпиши все классы эквивалентности отношения из задачи 12.
Во множестве X у нас джва класса эквивалентности A = {<a,b>, <b,c>, <c,a>} и B = {<d,e>, <e,f>, <f,a>};
> Задача 14. Перепиши фактор-множество из задачи 13, используя в качестве символа множества какой-нибудь его элемент.
Фактор-множество X будет множество {a, d}, a участвует в отношении a~b, b~c, c~a, а d~e, e~f, f~d, т.о., a и d будут "репрезентативными" элементами, составляющими фактор можество множества X.
> Хинт: можно построить точки на картинке и эквивалентные точки соединить линиями, а потом просто описать полученное.
Это полезный хинт, благодарю.
Кстати, я опять наверно глупость какую-то спрошу, но симметричность и рефлексивность как бы подразумеваются, если я понял правильно? Или, вероятно, блее правильно было бы сказать так: для класса эквивалентности A = {<a,b>, <b,c>, <c,a>} если некое a~b и b~c и c~a, то след-но b~a и a~a (если a=b, b=c, c=a) и тут естественным образом получается уже и симметричность и рефлексивность.
> Это вовсе и не абстрактное, тут конкретные буковки есть же, которые даже можно явно выписать.
Да, я понимаю же. Просто по мере роста градуса абстрактности немного дух захватывает, хе-хе.
Кстати, не посоветуешь ли чего-то для самостоятельной "работы"? Зорича мы немножко походя затронули, но наверно пока ещё рано плотно за него браться? Френкель-Бар-Хиллел - это вовсе для общего знакомства, онли, я так понел. Может что-то ещё?

Вы очень бобр, сэнсэй :3 Уж не знаю, какими мотивами ты руководствуешься, но хотел бы поблагодарить за то, что тратишь своё время на меня да ещё и на более-менее регулярной основе и за ободрения, конечно, тоже.

Альзо, с наступющим жэ и всего самого лучшего. Надеюсь на продолжение плодотворного сотрудничества и в грядущем году, как ныне модно говорить. Во~т.

с:ногами считалось битва хвоста королевы
>> No.134538 Reply
File: листок1.png
Png, 156.72 KB, 641×882
edit Find source with google Find source with iqdb
листок1.png
File: листок2.png
Png, 171.49 KB, 631×939
edit Find source with google Find source with iqdb
листок2.png
File: листок3.png
Png, 59.82 KB, 639×332
edit Find source with google Find source with iqdb
листок3.png
File: Grothendieck-smal...
Png, 22.86 KB, 557×324
edit Find source with google Find source with iqdb
Grothendieck-small.png

>>134519
> Некоторое X
Туда ещё нужно явно добавить <a,a>, <b,b> и прочее. Ещё нужно добавить пары для симметричности. То есть

{<a,b>, <b,c>, <c,a>, <d,e>, <e,f>, <f,d>,
<a,a>, <b,b>, ...
<b,a>, <c,b>, <a,c>, ...
}

Но ок, всё засчитано.
> "репрезентативными" элементами
Они называются "представителями класса", или просто представителями.
> симметричность и рефлексивность как бы подразумеваются
Это значит, что у тебя есть математическое мышление, и ты способен видеть теоремы. Весьма хорошо. Не надо характеризовать свои вопросы как "глупости".
В принципе, ты прав, рефлексивность можно было бы вывести из симметричности и транзитивности - но только если бы мы знали, что каждый элемент находится в паре с каким-то другим элементом, а это требование как раз эквивалентно требованию рефлексивности. То есть вот рассмотрим множество {a,b,c,d,e,f, w}. На нём можно задать отношение, построенное тобой, но оно не будет отношением эквивалентности, потому что никак не получится доказать, что w~w. Поэтому пару <w,w> придётся добавлять в отношение явно.
> Фактор-множество X будет множество {a, d}
Не совсем. Фактор-множеством будет { {a, b, c}, {d, e, f} }.
Ты записал это фактор-множество через представители.
> наверно пока ещё рано плотно за него браться
Нет, уже не рано, ты готов читать учебники. Просто я нахожу язык книги Зорича неудачным и нудным, на борьбу с этим языком при самостоятельном чтении приходится тратить неоправданно много сил, хотя более удачного учебника анализа на русском языке я не знаю (удачного, главным образом, из-за второго тома). Сначала давай разберём функции и мощность множеств (этот и следующий листочки), а потом поговорим об учебниках. "Урожаи и посевы" Гротендика читал? Если нет, то можешь пока что прочитать, это интересно.
> "работы"
Ты действительно работаешь, без всяких кавычек.

Так вот, функция. Это понятие оформилось не очень давно. Древние греки его не знали, например. Сама идея функциональной зависимости более-менее закрепилась в массовом сознании только в средневековье. Некоторые мысли на эту тему, слабые и размытые, высказывал Николай Орем в XIV веке. Окончательно в математику понятие функции ввёл только Лейбниц, опираясь на вышедшую сейчас из употребления идею постоянных и переменных количеств. С тех пор понятие функции продолжает эволюционировать.

В наши дни есть несколько не эквивалентных взглядов на функцию. Самый популярный взгляд - это, конечно, теоретико-множественный. Он оформился в первой половине XX века, опираясь на более ранние логико-философские концепции. Во второй половине XX века эти концепции были пересмотрены известными логиками, и, видимо, лет через сорок, когда наше поколение уйдёт со столпа титанов, теоретико-множественное рассмотрение функции в очередной раз изменится.

Чуть менее популярный взгляд - это рассмотрение функции в теоретико-категориальном ключе. Теория категорий даёт возможность рассмотреть функцию гораздо более абстрактно, чем теория множеств. Если в теории множеств функция вполне определяется своими значениями, то в теории категорий это, вообще говоря, не так.

Я изложу сейчас теоретико-множественное видение функции. С идеей функции ты, несомненно, знаком. В школе ты уже видел некоторые функции. Функцией в школе называется формула, путём которой одному числу сопоставляется другое. Этот школьный взгляд нельзя признать удовлетворительным, поскольку формула - это лишь строка символов, а ведь чувствуется, что функция есть нечто большее, чем просто буковки. Одна и та же функция может быть выражена несколькими формулами, например. В программировании ты, конечно, тоже встречался с функциями, но функции из программирования, в отличие от школьных, работают не только с числами. В программировании функция - это просто блок кода, на который наложены некоторые дополнительные ограничения - наличие имени, наличие возвращаемого значения, etc.

Можно усмотреть нечто общее между школьными и программистскими функциями, а именно идею сопоставления. Функцией называется то, что одному объекту (в общем случае n-ке объектов) сопоставляет другой объект. Это, вроде бы, настолько простая идея, что в разъяснениях не нуждается. Однако вот такие понятия, которые вроде бы очевидны, доставляют ментальный дискомфорт. В силу их очевидности о них толком ничего сказать нельзя, они просто есть. А это возмутительно. У всего должны быть какие-то свойства, обо всём должны быть какие-то факты. Каждый совсем не очевидный объект вроде "интер-универсальной теории Тейхмюллера: построение театров Ходжа" (с) бросает нашему сознанию вызов, но едва ли не больший вызов нам бросает каждый самоочевидный объект. Как он смеет быть самоочевидным?

Ладно, я отвлёкся. Итак, с помощью уже известных понятий множества и отношения надобно разъяснить, что значит, что "одному объекту сопоставляется другой". Что подразумевается под сопоставлением? Зафиксируем то, что нам уже известно.

Во-первых, объекты, которым функция будет сопоставлять другие объекты, в некотором смысле имеют одну природу. Это или любое из чисел, или любое из String и т.п.
Во-вторых, *каждому объекту сопоставляется один-единственный объект. Школьные функции доставляют одно число, программерские функции доставляют одно значение.

Вот на этих двух фактах и разовьём теорию. Первый факт означает, что объекты, которым функция будет что-то сопоставлять, образуют множество. Назовём его domain, обозначим буквой X. Тут же скажем, что объекты, которые имеют быть сопоставленными функцией, также образуют множество. Назовём его codomain, обозначим буквой Y.

Далее. Утверждение, что объекту x сопоставлен некоторый объект y, можно воспринимать как утверждение, что дана упорядоченная пара <x,y>. Раз есть упорядоченные пары, то есть и какое-то отношение. Второй факт естественно выразить путём наложения на возникшее вот отношение каких-то ограничений, то есть нам нужно ввести какой-то новый сорт отношений.

Назовём функциональным отношением вида <X,Y> отношение φ со следующими свойствами.
1. Для всякого элемента x ∈ X существует элемент y ∈ Y такой, что пара <x,y> принадлежит отношению φ.
2. Если x φ a и x φ b, то a = b.
Первое свойство означает, что любому объекту из X сопоставлен объект из Y. Второе свойство означает единственность сопоставленного объекта. Запись y=φ(x) означает, что пара <x,y> находится в отношении φ.

Функцией называется тройка [domain, codomain, функциональное отношение вида <domain, codomain>]. По ряду причин нельзя отождествить понятие функции и функционального отношения, обязательно нужна явная информация о domain и codomain.
Ну вот мы и истолковали понятие функции в понятиях множеств и отношений.

Итак, всегда, когда нам задана функция f, мы имеем право считать, что нам известны её domain, codomain, а также некоторое функциональное отношение. Мы можем записать это как f: X → Y, где X domain, Y codomain. Читается как "функция f отображает множество X на множество Y". Символ, кстати, довольно молодой, ему меньше века, раньше функции обозначали менее удачным символом. Когда додумались обозначать функции стрелочкой, случилась мини-революция.

Обычная школьная функция квадратный корень - это функция из вещественных чисел в вещественные числа. Символически √: ℝ → ℝ.
Функция void function1(integer, string, real) символически запишется как function1: integer × string × real → void.
Так называемые функции многих переменных - это просто функции, в которых domain есть декартово произведение.
Сложение - это функция двух аргументов, например. +: ℝ×ℝ → ℝ.

Слово функция в XX веке постепенно вытеснялось словом "отображение", но не вытеснилось до конца благодаря теории категорий, из-за которой функции сейчас часто называют морфизмами. Терминология здесь вообще довольно развитая. Функциями стараются называть отображения числовых множеств, операторами называют отображения нечисловых множеств. Функции, в которых domain и codomain совпадают, обычно называются преобразованиями. Например, string→string назовётся преобразованием множества string. Хотя это всё размыто, и кому как приятнее, тот так и говорит. Однако преподаватели этой страны иногда очень маразматично относятся к терминологии. Требуют обязательно различать функции и отображения, лол. Domain требуют называть "областью отправления", codomain "областью прибытия", дважды лол. Вся эта байда с терминологией случилась из-за приключившегося в СССР разоблачения "низкопоклонства перед Западом", а в этом году внезапно вспыхнула по-новой.

Немного обогатим нашу терминологию.
Квадратный корень, известный из школы, не может принимать отрицательных значений. Поэтому не любое число из ℝ может быть возвращено квадратным корнем. Мы введём термин, который такую ситуацию исключает. Сюръекция, или сюръективное отображение - это отображение f: X → Y такое, что для любого y ∈ Y существует такой x ∈ X, что f(x) = y. То есть сюръекция - это когда в каждый игрек отображается по крайней мере один икс.

В один и тот же y могут отображаться несколько разных элементов из X. Например, функция int func(string s){return 0;} и строке "foo", и строке "bar" сопоставит один и тот же ноль. Введём термин, запрещающий такую ситуацию. Инъекция, или инъективное отображение - это такое отображение f: X → Y, что если x1≠x2, то f(x1)≠f(x2). Разным иксам соответствуют разные игреки.

Наконец, биекцией, или взаимно-однозначным отображением, назовём отображение, которое и сюръективно, и инъективно. С помощью понятия биекции мы, следуя Кантору, скоро построим понятие мощности множеств.

Если y=f(x), то мы будем называть y образом x при отображении f, x - прообразом y. Все иксы, которые отображаются в y, мы назовём полным прообразом этого y. Прочувствуй разницу. Прообраз y - это какой-то элемент X, полный прообраз y - это какое-то подмножество X. Полный прообраз может быть пустым множеством, если в y не отображается ни один элемент. Например, полный прообраз -13 при описанном выше отображении √ пуст.

Вот листочек. Рисовать не обязательно. Как обычно, лучше поскорее, чем побольше.
> Альзо, с наступающим жэ и всего самого лучшего.
Спасибо, и тебя.
>> No.134605 Reply
Добрый анон, посоветуй окончательную книжку по общей алгебре для самостоятельного изучения just for fun.
>> No.134606 Reply
>>134605
> окончательную
Это как?
>> No.134608 Reply
>>133509
Анон, почему определитель СЛУ с n неизвестными и n уравнениями, для которого существует ненулевое решение, равен 0?
>> No.134611 Reply
Интересует представление логики в формальной системе. Как выбираются правила, аксиомы, чтобы формальная система предоставляла примеры, поддающиеся подразумеваемому при составлении системы толкованию?
>> No.134627 Reply
>>134606
ultimate в смысле.
>> No.134639 Reply
>>134608
> равен 0
Не равен же.
> почему
Потому что главное условие от СЛАУ с единственным решением — линейная независимость. А определитель не равен нулю только тогда, когда его строки (и столбцы) линейно независимы.
>> No.134685 Reply
>>134639
> единственным решением
В оригинальной постановки этого не было. И если определитель равен нулю, то значит у системы выпуклое множество решений. А нет решений
>> No.134688 Reply
>>134685
... только у неоднородной системы, у которой ранг расширенной матрицы не совпадает с рангом основной матрицы (теорема Кронекера-Капелли в помощь).
>> No.134709 Reply
File: Lewis-Fine-McDonnell.jpg
Jpg, 219.50 KB, 1024×768 - Click the image to expand
edit Find source with google Find source with iqdb
Lewis-Fine-McDonnell.jpg
Чики, добрач.

Я одинокий мудак, поэтому мою логику нужно переодически проверять на дефекты. В частности: пришел к выводу, что учить математику лучше по книгам, а не лекциям и семинарам. Например, у того же Горденцева в его «Алгебре» всё объясняется гораздо понятнее, чем на лекциях, полно упражнений. В конце каждого параграфа есть десятки заданий. Можно пойти на семинар, и мне дадут алгоритм решения для большинства заданий, а можно придумать алгоритм решения самому, отталкиваясь только от теории, и это лично мне гораздо интереснее. Можно читать Зорича, который всё пережевывает и кладет тебе в ротик и добавить Антидемидовича, а можно читать Натанзона и половину теорем, свойств и следствий находить самому. Опять таки, это ГОРАЗДО интереснее. Если какое-то задание долго не идет — попросить совета у анона или на каком-нибудь dxdy (к слову, реквестирую ресурсы, где можно просить помощи), или у кого-нибудь из преподов в вузе. Другая проблема — поиск годных ресурсов, ибо требования к качеству высокие, т.к. далеко не каждый учебник лучше лектора/семинариста.

И второе «в частности»: допустим, я хочу после бакалавриата отправиться в аспирантуру, и если там желание учиться не пропадет — заниматься исследованиями дальше. Но я немного нервничаю из-за того, что могу просто физически быть неспособным. Оказаться слишком тупым. Пока я пришел к таким идеям: чтоб понимать новое нужно быть сорт оф эрудитом: умение решать задачи сводится к задрачиванию уже знакомых алгоритмов до тех пор, пока они не станут естественными, как таблица умножения. В комплексных задачах помогает развитая рабочая память. Я читал доклады, из которых сделал вывод, что систематическая и высокая нагрузка на рабочую память улучшает ее. Чтоб создавать новое нужно хорошо знать теорию, эксперементировать, неплохо бы быть эрудитом и знать алгоритмы не связанные с математикой (типа определения электронной конфигурации атома), не бояться неизвестного.

Что думаете?
>> No.134714 Reply
>>134709
> эксперементировать
У тебя тут ошибонька. "Экспериментировать". Мне кажется, без фидбэка трудно. Не только в математике. Вообще в любой сфере знаний. Не обязательно, чтобы тебе разжёвывали, но очень важно, чтобы кто-то сказал "u doing it wrong". Книга не даст тебе фидбэка, потому что она только говорит, но не слушает. Если ты плотно засядешь на dxdy, то чем это будет отличаться от общения с живым преподавателем, которого можно спросить и получить ответ? Кроме того, что ты сам возводишь себе препятствие, при условии, что у тебя такой преподаватель есть. Если его нет, тогда, конечно, другой вопрос. Я так думаю.

%Мне кажется, эрудиция - более широкое понятие, чем знание алгоритмов. Это больше знание фактов вообще, безотносительно к их алгоритмичности.%%

А что спросить-то хотел?
>> No.134716 Reply
Есть уравнение "x + y + xy = n", где 'n' — натуральное число ∈ [0;+∞), а 'x' и 'y' переменные.
Нужно разработать оптимальный алгоритм, позволяющий определять все корни при любом 'n'. Достоверно известно что существует полное решение.
>> No.134724 Reply
>>134714
> А что спросить-то хотел
Хотел убедиться, что это не бред поехавшего.
>> No.134730 Reply
>>134716
x = (n-y)/(1+y)
Отдельно вариант y = -1, x - любое, n = -1 - откидывается, т.к. n - натуральное.
Если x и y действительные, то решений бесконечно много. Наверно в задаче подразумевается, что x и y - целые (может быть даже неотрицательные)?
>> No.134747 Reply
>>134724
А тебе есть дело?
>> No.134766 Reply
>>134538
> Задача 1
а) и г) задают отображение f:X → Y, в то время как в) - однозначно нет, одному x ∈ X соответствуют два y ∈ Y; б) - я в затруднении. По определению (каждому x ∈ X сопоставляет элемент y ∈ Y) б) не задаёт отображение. Но вообще говоря, как и в примере с квадратным корнем отображение f может быть задано на подмножестве множества X, так что я скорее склонен считать, что б) всё же задаёт легитимное отображение.
> Задача 2
{<7,0>, <8,0>, <9,0>}, {<7,0>, <8,0>, <9,1>},
{<7,0>, <8,1>, <9,0>}, {<7,0>, <8,1>, <9,1>},
{<7,1>, <8,0>, <9,0>}, {<7,1>, <8,0>, <9,1>},
{<7,1>, <8,1>, <9,0>}, {<7,1>, <8,1>, <9,1>}

Итого, 8 различных вариантов (хм, 2³ возможных перестановок).
> Grothendieck-smal.png
Внезапно, лол. Да, я его сейчас начал читать по твоей рекомендации. Если честно, чтиво не из самых лёгких и приятных. Во-первых, автор много говорит о вещах не вполне понятных, как то: топосы, схемы, когомологии. Без понимания таких моментов сложнее проникнуться тем, что же хотел сказать автор. Во-вторых, страсть мэтра к сноскам не добавляет тексту простоты. Но вообще любопытно. Не думал, что в топологии, алгебре и геометрии всё (было) так драматично.
> Ладно, я отвлёкся.
Здорово. Это было интересно, хотя местами не совсем понятно
> "интер-универсальной теории Тейхмюллера: построение театров Ходжа" (с)
>> No.134768 Reply
>>134730 продолжаю:
x = (n-y)/(1+y) = (n+1-1-y)/(1+y) = (n+1)/(1+y) - 1
Таким образом, для целых x и y всё свелось к задаче поиска делителей числа n+1, что уже проще нагуглить.
>> No.134772 Reply
Продолжаю >>134766:
> Задача 3
Образ множества f[{0, 3}]:
а) {18, 5}; б) {18, 5}; в) {2, 7};

Образ множества f[{1, 3, 4}]:
а) {7, 5}; б) {2, 5, 7}; в) {7, 2};

Полный прообраз элемента f⁻¹(2):
а) Ø; б) {1}; в) {0, 4};

Прообраз множества f⁻¹[{2, 5]]:
а) {3, 4}; б) {1, 3}; в) {0, 4};

Прообраз множества f⁻¹[{5, 18}]:
а) {3, 4, 0}; б) {3, 0}; в) Ø.
>> No.134773 Reply
>>134766
> б) не задаёт отображение
Да, по данному определению это так.
> в примере с квадратным корнем
Я там просто не всё написал, что собирался. Хотел рассказать о более общем понятии - отношения, которое может быть полным и не полным слева/справа - но не рассказал, ну да и не страшно. Отношение, соответствующее квадратному корню, задумывалось как пример отношения, которое не полно слева. Извиняюсь, что не доредактировал пост. Энивей, работать надо с тем определением, которое я дал. Когда говорят об отображении, почти всегда подразумевают полноту слева.
> б) всё же задаёт легитимное отображение
На это можно смотреть и так, да. Это называется "частично определённая функция", partial function, и у этого даже есть кое-какая теория. Обычно предполагается, что все функции вполне определённые. Просто кто-то однажды может придираться к словесным оборотам вроде "функция может быть не определена в точке", дескать, какая же это функция. Частично определенная, вот какая.
> Итого, 8 различных вариантов
Да.
> хм, 2³ возможных перестановок
Очень хорошо, что ты это заметил. Это пригодится.
> всё (было) так драматично
You know nothing, Jon Snow.
> о вещах не вполне понятных, как то: топосы, схемы, когомологии
Гротендик чувствовал эти вещи совсем не так, как их ощущаю, например, я. Он, как мог, передал своё представление, но из-за несовершенства языка у него это получилось не слишком удачно. А под конец жизни, когда он уже жил отшельником в районе Пиренеев и общался там с каким-то божеством, он и вовсе запретил публиковать свои работы, так что понять его уже не получится. Щито поделать.
> хотя местами не совсем понятно
А это есть один такой Мотидзуки в Японии.
http://habrahabr.ru/post/183374/ - пост жёлтый, но суть верна.

>>134772
Ок.
>> No.134774 Reply
>>134773
Что я, кажется, всё же сумел ухватить из сути, Гротендик говорит об обобщении "непрерывных" и "дискретных" структур. Т.е., гибрид алгебры и геометрии с довольно интересными следствиями на фоне тех свойств этих дисциплин, о которых лично я как-то никогда и не задумывался "дискретность" алгебры и "непрерывность" геометрии? Как-то так наверно. Но попытавшись немного погуглить обо всём вот этом, я понел, что пока не стоит это трогать, хе-хе. И тем не менее, как я сказал, интересно, хотя и далеко от того, чтобы назвать этот труд удобоваримым. В оригинале там вроде вообще 1000+ страниц на французском, кажется?
>> No.134775 Reply
>>134774
Мы эти вещи разберём в своё время, определения-то не очень сложные. Топологию так и вообще очень скоро введём, прямо перед понятием предела.
> В оригинале там вроде вообще 1000+ страниц на французском, кажется?
Ага.
>> No.134780 Reply
>>134747
Да. Логика либо есть, либо её нет, и для дальнейшей деятельности мне важно понимать есть ли она в моей голове, или пора бы эту голову перезагрузить.
>> No.134786 Reply
Elementary Analysis это и есть матан? Еще есть Real analysis, что это за зверь?
>> No.134790 Reply
>>134786
Разве матан (именно собственно математический анализ со всякими там limits, integrals, differentials) - это не calculus? Elementary - это может начала/введение в calculus? На самом деле ХЗ, в общем.
>> No.134791 Reply
>> No.134802 Reply
>>134786
Да, это и есть матан. На пальцах объясняются концепции предела, непрерывности, дифференцирования и интегрирования.
Real Analysis концентрируется на изучении вещественных чисел. Этот курс, в принципе, не слишком сильно отличается от Elementary Analysis, разве что его читают поабстрактнее и обычно добавляют плюшки вроде теоремы Витали, сингулярных интегралов и классификаций Бэра.
Complex Analysis изучает голоморфные и мероморфные функции. Главный геометрический объект - риманова поверхность.
enwiki://Glossary_of_areas_of_mathematics - про остальные analysis можно посмотреть тут.

>>134790
Calculus - часть Analysis, концентрирующаяся на вычислениях.
>> No.134823 Reply
>>134802
> Calculus - часть Analysis, концентрирующаяся на вычислениях.
Но книжки и курсы по обычному матану имеют слово Calculus в названии. Или какие вычисления имеются в виду?
>> No.134826 Reply
>>134823
> Или какие вычисления имеются в виду?
Всякие там "взять неопределенный интеграл", "найти предел", "вычислить значение сотой производной данной функции в нуле", etc.
Большая часть известного нам всем младшекурсного матана - это именно calculus.
>> No.134843 Reply
>>134775
Кто "мы"?
[i]мимокрок[/i]
>> No.134844 Reply
>>134843
Святая Толстота и его ситх-ученик.
>> No.134845 Reply
>>134826
>>134826
А, мне почему-то подумалось, что ты про вычметы - прошу прощения, тогда все так.
>> No.134847 Reply
>>134844
лол
>> No.134850 Reply
Знаю, что банально, но всё же подскажите учебник/самоучитель для того, чтобы подогнать знание математики к высокому для гос. экзаменов в конце 11-го класса.
>> No.134851 Reply
>> No.134863 Reply
Как изобразить y=x-a? Здесь а - параметр.
>> No.134864 Reply
>>134863
Параметр - это просто аргумент функции. Тебе требуется изобразить функцию двух переменных вида z = (x,y). В твоём конкретном случае это плоскость z = x-y.
Если же имеется в виду, что a - это какое-то конкретное число, но не переменная, то тебе требуется просто нарисовать прямую типа http://www.wolframalpha.com/input/?i=y+%3D+x-3
Просто сдвигаешь по оси y на a.
>> No.134881 Reply
Аноны, есть какая-нибудь литература для чайников, поясняющая за топологию?
>> No.134882 Reply
>>134881
> для чайников
Журнал "Квант" и его Библиотечка. Конкретные выпуски не помню.
>> No.134885 Reply
>>134850
Сканави же.
>> No.134886 Reply
>>134864
Под параметром обычно таки понимают фиксированное произвольное число, так что наверняка про прямую спрашивал анон.
>> No.134891 Reply
>>134886 а как же циклойды?
>> No.134899 Reply
>>134891
Как ты умудрился поставить в этом слове й? Нет, мне правда интересно, это не насмешка.
>> No.134919 Reply
>>134899
Циклойды, кардойды, андройды.
>> No.134920 Reply
>>134919
Откуда там "й"?
>> No.134932 Reply
>>134881
Смотря какую, есть книжка
Васильева: топология для младшекурсников. В любом случае, если там слишком много (а нужно задрачивание до дыр компактности, например), её всё равно стоит прочитать, чтобы понять куда эта наука приводит, мне кажется так будет легче. Тем более - книжка тонкая.
>> No.134935 Reply
Математики, у меня вопрос по поводу континуум-гипотезы. Её отрицание предполагает существование множеств мощнее N но менее мощных чем континуум. Однако, так как континуум-гипотеза независима от ZFC, конструктивных построений данных множеств нет (иначе бы они опровергали континуум-гипотезу).
Таким образом, существуют множества, для которых в принципе невозможны никакие конструктивные построения. Как же так?
>> No.134936 Reply
>>134935
Брауэр, ты опять выходишь на связь?
>> No.134945 Reply
>>134936
шта
>> No.134946 Reply
>> No.134959 Reply
File: Untitled-1.jpg
Jpg, 247.17 KB, 793×1122 - Click the image to expand
edit Find source with google Find source with iqdb
Untitled-1.jpg
>>134773
Ученик-ситх снова выходит на связь, лол. В общем, шаманство с разметкой и символами из юникода меня немного утомили, поэтому я отрендерил свою писанину в джипег. Надеюсь, это нормально.

Задача 4 - это скрежет зубовный. Ибо воистину, как эти объекты смеют быть самоочевидными. No pun intended. Seriously

Кстати, о бурбакизме. Я так понял, не все считают труды Бурбаки одинаково полезными? И ещё, я так и не понял, что же с Гротендиком. Он действительно, буквально поехал на старости лет? Хотя достоверной информации мало. Что вполне логично, если учесть его неконтактность в последние годы. Но хоть после пресловутых 70-х он продолжал заниматься математикой, тем не менее, его библиография какая-то довольно куцая. Всё это несколько удивляет на фоне его статуса. Пусть и не совсем официального. Непонятно, в общем. Впрочем, ето оффтоп.
>> No.134998 Reply
File: множества.png
Png, 67.33 KB, 800×1622 - Click the image to expand
edit Find source with google Find source with iqdb
множества.png
>>134959
> это скрежет зубовный
Как только ты замечаешь, что занимаешься нудной и непонятной символодрочкой, ты должен говорить себе: "Стоп, тут что-то не так, нужно найти что-нибудь полегче и поубедительнее". Доказательства пишут не для того, чтобы любой ценой убедить собеседника, а для того, чтобы доказать истинность факта самому себе. А символодрочка может показаться убедительной очень редко. Смысл доказывания теоремы не в том, чтобы записать доказательство по всем бюрократическим канонам, а в том, чтобы сказать "да, этот факт верен, и вот мои аргументы". Руссо в "Исповеди" писал, что не верил доказанной им самим формуле "квадрат бинома равен сумме квадратов его членов и их удвоенному произведению", то есть (a+b)^2 = a^2 + 2ab + b^2, до тех пор, пока не начертил фигуры. И это нормально, доказательство должно убеждать в первую очередь доказывающего. Коши независимо изобрёл алгебраическую запись комплексных чисел, но долгое время метался и то публиковал её, то отвергал её - ибо не верил тому, что изобрёл.

Существуют люди, которые считают, что доказывать теоремы нужно обязательно по Строгим Правилам: записывать посылку теоремы, писать строчку кракозябр, писать ещё одну строчку кракозябр, писать больше кракозябр, ещё больше... и только потом записывать вывод. Чем больше кракозябр в доказательстве, тем более ценным оно представляется таким людям, а вывод словно бы и не важен. Таких людей очень много.

Однако доказательства проводят не затем, чтобы записывать кракозябры. Доказательство проводят затем, чтобы доказать факт. Вообще, если ты идёшь к какой-то цели, то достижение цели гораздо важнее соблюдения всех традиционных ритуальных шагов. Если можно прийти к цели более лёгким путём - иди по этому пути. Цель битвы на мечах не в том, чтобы постукать мечами друг о друга. Цель битвы - убить врага. Процитирую Фейнмана:

Примерно в то же время мой двоюродный брат, который был тремя годами старше, учился в средней школе. Ему с трудом давалась алгебра, поэтому к нему приходил домашний учитель. Мне разрешали сидеть в уголке, когда учитель пытался научить моего брата алгебре. Я слышал, как он рассказывает об x.
Я сказал брату: «Что ты пытаешься сделать?»
– Я пытаюсь найти, чему равен х в уравнении 2х + 7 = 15. Я говорю: «Ты имеешь в виду 4».
– Да, но ты применил арифметику. А его нужно найти с помощью алгебры.
К счастью, я изучил алгебру, не ходя в школу. На чердаке я нашел старый учебник алгебры, принадлежавший моей тете, и понял, что вся идея состоит в том, чтобы найти х – неважно как. Я не видел разницы в том, чтобы найти его «с помощью арифметики» или «с помощью алгебры». «Сделать это с помощью алгебры» означало взять набор правил, которые, если им слепо следовать, могут дать ответ: «вычти 7 из обеих частей уравнения; если у тебя есть множитель, то раздели на него обе части», – и так далее – ряд шагов, с помощью которых можно получить ответ, если не понимаешь, что пытаешься сделать. Правила изобрели, чтобы все дети, которые должны изучать алгебру, могли сдать экзамен. И именно поэтому моему брату никак не давалась алгебра.

Цель, которую преследовал брат Фейнмана, была не в том, чтобы найти x. Его цель была в том, чтобы применить неудобные Правила, потому что дядя учитель требует их применить.

Если ты будешь почаще спрашивать, нет ли более простого способа доказать теоремку, нежели грубой силой, то сэкономишь себе кучу времени.

г) Пикрелейтед. Пересечение множеств A - пустое множество, образ пустого множества - пустое множество, но пересечение образов A - не пустое множество. Так что там не равенство, а только включение. f(A⋂B) ⊂ f(A)⋂f(B).
е) Опять же, см. пикрелейтед.
ж) Мысленно вдвинь множества B1 и B2 одно в другое - вот и контрпример.

Ты, кстати, до сих пор используешь предикаты не совсем в общепринятом смысле. Предположу, что ты мыслишь так.
1. Есть какие-то строки символов. Они могут быть либо истинными, либо ложными.
2. Есть какая-то функция, обозначаемая буквой P, которая принимает на вход строку и выводит либо истину, либо ложь.

Та идея, которую ты здесь мыслишь под P, называется "классификатор подобъектов". Он используется в теории категорий для образования так называемого логического топоса. Топосы позволяют строить очень много логик; есть даже гипотеза, что любое мыслимое человеком понятие может быть выражено на языке логических топосов. Эту идею запомни, она тебе несомненно пригодится.

Однако с предикатами всё проще. P(x) есть просто формальное обозначение строки букв, в которой нас особым образом интересует буква x. То есть неверно, что P - это какая-то зарезервированная буква, это просто имя строки. Никакой специальной функции, которая проверяет строки на истинность или ложность, в языке предикатов явным образом не вводится. Чтобы выделить множество четных чисел из множества целых чисел, нам нужно записать не {x ∈ ℤ; P(x = 2k)}, а просто {x ∈ ℤ; x = 2k}.
> бурбакизме
Бурбакизм - это слово, которое активно форсил великий советский математик Арнольд. Арнольд был советским человеком с еврейскими корнями - то есть был недоброй и обидчивой личностью с очень хорошей памятью. Невзлюбил бурбаков он по личным причинам, не буду сейчас их пересказывать, ибо некультурно. Вратце - Арнольда унизили и обидели. Поскольку математиком Арнольд всё-таки был великим, влияния у него было много, из-за чего в СССР и отчасти в РФ к осуждению бурбакизма присоединились многие люди. Однако это не было каким-то хардкорным осуждением - никого не убили, насколько я знаю. Просто были всякие конференции и интеллектуальные, интеллигентные срачи. Ближе к концу жизни Арнольд то ли сам попал под грузовик, то ли его кто-то под него скинул, в общем, Арнольд получил тяжёлую черепно-мозговую травму, от которой не оправился до конца жизни. Эта травма была очень тяжёлой, ему пришлось заново учиться говорить. Последствия тоже были печальными, Арнольд начал нести всякую ахинею про Гермеса Трисмегиста и изумрудную скрижаль, а также бороться с компьютерами. Впрочем, арнольдовская критика бурбаков по большей части была справедливой.
> Я так понял, не все считают труды Бурбаки одинаково полезными?
Труды самого Бурбаки нужно отделять от трудов участников группы Бурбаки. Труды Бурбаки - это большой, многотомный трактат, в котором на основе философских воззрений начала XX века развивается некая формальная теория, включающая в себя большую часть того, что сейчас считается стандартным набором знаний математика. Трактат написан очень абстрактным языком и разрушает многие устои, из-за чего стал эпическим мемом. Однако главная ценность этого трактата, я бы сказал, теологическая, реальной пользы от него немного. В качестве учебника его использовать нельзя, например. К тому же на русский язык этот трактат переведён как попало. Я нашёл в первой главе первого тома в одном из Критериев, уже не помню его номер, ошибку перевода. Это действительно ошибка перевода, во французской версии книги этой ошибки нет. Однако гугл показал, что никто, кроме меня, из обитающих в сети математиков и около-математиков эту ошибку не нашёл, что в общем-то позволяет сделать вывод о популярности Трактата в этой стране.

http://absurdopedia.net/wiki/Бурбаки
ruwiki://Бурбаки
> И ещё, я так и не понял, что же с Гротендиком. Он действительно, буквально поехал на старости лет?
Возможно. Сумасшествие - это нормально для хороших математиков. У Кантора был маниакально-депрессивный психоз, Гедель боялся холодильников, про Нэша известное попсовое кино сняли.
Воеводский вот с астралом общается, http://baaltii1.livejournal.com/198675.html и http://baaltii1.livejournal.com/200269.html . Но нельзя исключать возможность, что Гротендик просто был поэтом. В конце концов, он же сын своих родителей, ruwiki://Шапиро,_Александр_Петрович
>> No.135007 Reply
>>134998
Да, я догадывался, что суть доказательства не в количестве кракозябров. Просто я попытался достичь цели отталкиваясь от формальных свойств объектов и мне показалось, что тут, соответственно, будет уместен формальный язык кракозябров. А много их скорее потому, что надо было сделать это понятным прежде всего для себя. Отсюда вот эти промежуточные записи. Но что-то пошло не так. И возможно я даже уже вижу, что. Но для этого хотел уточнить, а почему в г) и е) подразумевается, что A₁ ⋂ A₂ = ∅? Я наверно снова где-то туплю, но мне не очевидно, что 1) A₁, A₂ ⊂ X и 2) A₁ ⋂ A₂ = ∅. 1) никак не влечёт за собой с неизбежностью 2). Кажется. Возможно тогда более правильно f[A ⋂ B] ⊆ f[A] ⋂ f[B]?
> У Кантора был маниакально-депрессивный психоз
Вряд ли мания или депрессия способствуют продуктивной деятельности. Но кто знает. Вот о холодильниках и Гёделе не доводилось слышать.
>> No.135008 Reply
>>135007
> будет уместен формальный язык кракозябров
Вообще он уместен, но он не слишком-то нагляден. Картинка ничуть не хуже.
> подразумевается
Не подразумевается, это лишь один из возможных вариантов. Можно на той картинке мысленно передвинуть A каким угодно образом, в том числе сделать их пересекающимися и даже совпадающими. Но доказывать-то надо для общего случая, а в общем случае A могут и не пересечься.
> ⊆
Да, это я и имел в виду, конечно. Кстати, эти факты из номера 4 ты доказываешь затем, чтобы потом их использовать при изучении топологии, это не какое-то там бессмысленное упражнение. У нас будут специальные множества, называющиеся открытыми, и мы будем внимательно рассматривать их образы, для этого все эти теоремки и пригодятся.
> о холодильниках и Гёделе
ruwiki://Гёдель,_Курт
http://lj.rossia.org/users/tiphareth/768813.html
Вербицкий, кстати, тоже поехавший.
>> No.135010 Reply
>>135008
О, спасибо. Тогда, думаю, всё более или менее встало на свои места.
> Картинка ничуть не хуже.
Не спорю и даже не очень люблю книги без картинок по этой причине. Шутка, конечно. Но в каждой шутке есть доля шутки, как говорится. Многие вещи действительно гораздо проще воспринимать визуально, если можно так выразиться - во плоти и крови, а не как набор формальных свойств. Но в данном случае мне кажется, что сперва нужно ужиться со свойствами, чтобы рисовать правильные картинки. С простыми множествами было проще. Диаграммы Эйлера/Венна для "простых" множеств действительно нагляднее. Но простые множества сами по себе достаточно "интуитивны", если позволительно так сказать.
> Вербицкий, кстати, тоже поехавший.
Миша - это вообще своеобразный мем, хе-хе. О нём наверно кто только не слышал на АИБ Или мне только так кажется? Но всё равно личность примечательная
>> No.135013 Reply
File: marsh-mira.jpg
Jpg, 78.13 KB, 453×604 - Click the image to expand
edit Find source with google Find source with iqdb
marsh-mira.jpg
>>135010
> О нём наверно кто только не слышал на АИБ
На самом деле уже мало кто. Всё течёт, время идёт.
>> No.135014 Reply
>>135008
Вот кстати. Может аналогия не слишком удачная, но вот именно о картинках. Так называемые проекции многообразий Калаби-Яу стали таким же мемом, как кот Шрёдингера. Много кто о них знает, но мало кто понимает суть™. Я и сам не исключение, потому что картинки кренделей Калаби-Яу ничего мне не говорят о том, что это за такие таинственные многообразия. Тот самый случай, как мне кажется, когда без знания свойств картинка - это всего лишь картинка.
>> No.135015 Reply
>>135014
> мало кто понимает суть
Очень мало. Я вот только припоминаю, что они имеют какое-то отношение к М-теории. И на этом всё.
>> No.135018 Reply
>>135014
Я в будущем дам ITT небольшие листочки на все понятия, нужные для определения этих многообразий если будет на то Воля Божья™, иншалла. В принципе, определение весьма простое, даже если обойтись без теории категорий. Но ruwiki://Экстенсионал этого понятия интересующимся анонам раскрывать придётся самостоятельно, по литературе, теорем об этом объекте я знаю мало.
>> No.135020 Reply
>>135018
Да Аллах с ними. Хотя, в принципе, выглядит занятно и таинственно, а потому интересно. Ну и модные теории вроде М и суперструн, опять же, ага. Надо почитать The shape of inner space для мотивации, хе-хе
>> No.135021 Reply
File: library.jpg
Jpg, 82.05 KB, 500×300 - Click the image to expand
edit Find source with google Find source with iqdb
library.jpg
>>135020
Я и так собирался это сделать, понятия-то важные.
>> No.135037 Reply
>>134998
Ты очень много знаешь. Аспирант, постдок, или просто любишь математику? Математик или философ? В любом случае спасибо за твои посты. Их всегда интересно читать.
>> No.135041 Reply
>>134998
> – Я пытаюсь найти, чему равен х в уравнении 2х + 7 = 15. Я говорю: «Ты имеешь в виду 4».
> – Да, но ты применил арифметику. А его нужно найти с помощью алгебры.
> ...
> Цель, которую преследовал брат Фейнмана, была не в том, чтобы найти x. Его цель была в том, чтобы применить неудобные Правила, потому что дядя учитель требует их применить.
Но ведь вроде бы очевидно, что дядя учитель требует их применить, чтобы научить детей общему принципу, который работает для любых чисел, а конкретное уравнение тут не так уж и важно.

В общем совершенно не понял, что имелось в виду.
>> No.135051 Reply
Есть ли какая-нибудь формальная и полностью консистентная математическая нотация (то есть чтобы ее можно было распарсить прогой при желании), пригодная для написания конспектов? Испытываю жуткий дискомфорт от возможных разночтений.
>> No.135053 Reply
File: задача.png
Png, 158.44 KB, 1304×881 - Click the image to expand
edit Find source with google Find source with iqdb
задача.png
>>135037
Просто анонимус.

>>135041
http://nbspace.ru/math/

>>135051
Есть программы распознавания математического ввода, например "Панель математического ввода", входящая в стандартную поставку винды. Они иногда делают ошибки. Полностью безошибочной распознавалки нет.
>> No.135055 Reply
>>135053
> Есть программы распознавания математического ввода, например "Панель математического ввода", входящая в стандартную поставку винды. Они иногда делают ошибки. Полностью безошибочной распознавалки нет.
Я, видимо, недостаточно ясно выразился. Вопрос был не о программе для распознавания ввода, а о нотации для использования в собственное удовольствие, с условием, что нотация такова должна быть, чтобы даже компьютер ее мог распарсить, буде на то чье-либо желание. В общепринятой нотации кроме множества разночтений есть еще и множество путей сказать одно и то же. Вместо того, чтобы пытаться изобрести для себя какие-то собственные (более строгие) правила ее использования, я надеюсь на совет мудрого анонимуса, могущего предложить по теме нечто готовое, и, быть может, даже используемое пусть не всеми, но хотя бы в определенных кругах.
>> No.135056 Reply
>>135055
Есть языки вот этих систем: enwiki://Proof_assistant , но они все запутанные, и ни один нельзя использовать с удовольствием.
> есть еще и множество путей сказать одно и то же
Это не баг, это фича.
>> No.135058 Reply
>>135056
Так ведь я знаю про пруф-ассистанты, потому и сделал акцент на возможности писать конспекты в такой нотации. То бишь от руки и быстро.
> Это не баг, это фича.
А меня раздражает и мешает сосредоточиться на смысле. К тому же что ж это за фича, которая не несет дополнительной выгоды?
>> No.135061 Reply
>>134998
> Сумасшествие
> маниакально-депрессивный психоз
facepalm.jpg.
>> No.135062 Reply
File: fishglobe_by_rishumisu-d6bmz0q.png
Png, 897.82 KB, 800×1166 - Click the image to expand
edit Find source with google Find source with iqdb
fishglobe_by_rishumisu-d6bmz0q.png
>>135058
Я не встречался с тем из этого, что могло бы быть удобным для конспектов. Спрашивал, но мне сказали, что ничего подходящего нет. Но тщательно я не искал, кто-нибудь другой может знать.
Смысл - слишком неясная категория. Есть более конкретные штуки - интенсионал и экстенсионал. Возможность выбирать из интенсионала разные наборы признаков, определяющие один и тот же экстенсионал, даёт возможность совершенствовать язык - то есть даёт главную выгоду любого теоретизирования.
>> No.135073 Reply
>>135062
Ок, я имел в виду более простые кейсы (каждый vs любой и подобное).
>> No.135116 Reply
>>135115
Еноты - няшки.
>> No.135117 Reply
File: d372afc46b19.jpg
Jpg, 88.13 KB, 640×480 - Click the image to expand
edit Find source with google Find source with iqdb
d372afc46b19.jpg
>>135116
Да.
>> No.135157 Reply
File: 5912988835_8a52ee3ce2_o.jpg
Jpg, 19.82 KB, 540×374 - Click the image to expand
edit Find source with google Find source with iqdb
5912988835_8a52ee3ce2_o.jpg
Анончик, скажи мне, что я справлюсь, а то я очень нерничаю. Умом понимаю, что вполне могу затащить, если буду стараться, но всё равно страшно.
>> No.135159 Reply
>>135157
Ты справишься, бокскаттер-кун.
>> No.135160 Reply
>>135159
Спасибо!!!
>> No.135199 Reply
File: 111.jpg
Jpg, 29.69 KB, 819×233 - Click the image to expand
edit Find source with google Find source with iqdb
111.jpg
Помогите, молю!
Откуда они берут закон изменения скорости?
>> No.135202 Reply
>>135199
Эм, взяли производную от x(t)? Т.е., v(t) = x'(t). Производная второго порядка дала бы уже ускорение, например. Нет?
>> No.135203 Reply
>>135199
Производная от координаты же.
>> No.135204 Reply
>>135199
Я, конечно, не эксперт, но, по-видимому, из определения.
ruwiki://Скорость
>> No.135206 Reply
File: slowpoke.png
Png, 11.90 KB, 312×295 - Click the image to expand
edit Find source with google Find source with iqdb
slowpoke.png
>>135202
>>135203
>>135204
Спасибо вам большое! А почему в производной координаты мы получаем t, а не 1/2*t?
>> No.135207 Reply
>>135206
Я, конечно, не эксперт, но, по-видимому, из определения.
ruwiki://Производная_(математика)
>> No.135208 Reply
>>135206
См. правила дифференцирования там, где указал >>135207. В частности, случай (f/g)'.
>> No.135209 Reply
>>135208
Ме. Дико извиняюсь. Не там, а в ruwiki://Производная_функции на самом деле. Мне почему-то подумалось, что обе ссылки ведут на одну статью.

фикс
>> No.135220 Reply
>>135206
> мы получаем t, а не 1/2*t?
Ну это пиздец, товарищи. Извините
>> No.135226 Reply
File: Untitled.png
Png, 2.73 KB, 451×33 - Click the image to expand
edit Find source with google Find source with iqdb
Untitled.png
Как подобные решаются?
>> No.135230 Reply
Поясните по хардкору, как на практике используются производные, тригонометрия и прочие мифические штуки.
>> No.135232 Reply
>>135230>>134836 две трети книги посвящены именно этому. коротко: задачи математического моделирования физических явлений
>> No.135235 Reply
>>135232
А пределы тоже преследуют эту цель? В голове не укладывается.
>> No.135238 Reply
>>135235 пределы - инструмент исследования. далеко не всегда очевидно, что будет, если произвольно шатать аргументы функции. с другой стороны красивые функции существуют только в учебниках, а на практике не всегда возможно подставляя значение аргумента узнать, чему равно значение функции. функции в интересующей точке может и не существовать, или она слеплена из кусков нескольких разных
>> No.135244 Reply
>>135226
Навскидку, это сравнение разлагается на сравнение по модулю пять и сравнение по модулю шесть: чтобы выражение от х в левой части делилось на 30, необходимо и достаточно, чтобы оно делилось и на пять, и на шесть. Эти сравнения решаются перебором, дальше применяется китайская теорема об остатках.
Более правильный и быстрый метод решения(который я не помню) изложен, скажем, в книжке Нестеренко "теория чисел".
>> No.135250 Reply
>>135230
Никак. Бред сумасшедшего.
>> No.135262 Reply
>>135230
Если ты занимаешься бухгалтерией, то можешь проинтегрировать увеличение объёма оборотных средств с течением времени например.
Но это всё выполняется машинами. Намного быстрее и эффективнее, если уметь пользоваться тем же экселем.

Ну и так, на практике можно забивать все эти интегралы/производные/диффуры в маткад/матлаб и моделировать сложные процессы.
Мы вот моделировали давление флюидов в толще пласта на разном количестве скважин. Температуру там считали, при равной скорости выкачивания нефти. Интересно в общем.

Правда это не хардкорный матан, а эмпирические формулы в сферическо-вакуумном виде, без учёта целой кучи факторов, изменяющегося напора, целого ряда технических трудностей. Но кому это интересно?
>> No.135264 Reply
File: doc135744478_274751364.gif
Gif, 2198.07 KB, 400×300 - Click the image to expand
edit Find source with google Find source with iqdb
doc135744478_274751364.gif
>>135244
Я из этого вообще ничего не понимаю... У Несетеренко ничего похожего тоже на нашел. А экзамен уже очень близко. Научите, пожалуйста...
>> No.135269 Reply
File: 20150113_120350.jpg
Jpg, 2386.13 KB, 2448×3264
edit Find source with google Find source with iqdb
20150113_120350.jpg
File: 20150113_120116.jpg
Jpg, 2271.21 KB, 2448×3264
edit Find source with google Find source with iqdb
20150113_120116.jpg

>>135264
Я как и тот товарищ, ничего конкретного тебе не могу сказать, могу только привести решение конкретно того уравнения. Как решать, чтобы прямо многочлен сколь угодно большой степени, я не знаю. Другое дело, что сравнительно небольшую степень по сравнительно небольшому модулю можно решить как я показал, ну, или если у тебя уравнение вида x^n = a (mod m) и так хорошо совпало, что есть первообразный корень, ты знаешь что делать.
>> No.135271 Reply
>>135264
Можешь скинуть сюда программу твоего экзамена? Я немного знаю литературу по теории чисел, могу подсказать.
>> No.135274 Reply
>>135264
> У Несетеренко ничего похожего тоже на нашел.
Плохо искал. Алгоритм решения сравнений по простому модулю начинается со стр. 118, по составному - стр. 123 в издании 2008 года.
>> No.135297 Reply
File: 3.png
Png, 129.08 KB, 958×880
edit Find source with google Find source with iqdb
3.png
File: 2.png
Png, 64.85 KB, 938×858
edit Find source with google Find source with iqdb
2.png
File: 1.png
Png, 30.91 KB, 943×265
edit Find source with google Find source with iqdb
1.png
File: Untitled.png
Png, 54.81 KB, 946×494
edit Find source with google Find source with iqdb
Untitled.png

>>135269
Хм. Ну, теперь я как минимум приблизительно знаю, чего я не знаю...
>>135271
Увы, нету. Могу только задачки семинаров и к.р. скинуть, и надеяться, что что-нибудь такое и будет.
>>135274
И правда. Я слепой значит.

Спасибо за ответы, аноны...
>> No.135305 Reply
Кхак я понял, весь курс школьной математики можно по Khan Academy изучить, если знать английский? Просто там реально объясняют раз в тысячу лучше, чем все учебники мира.
>> No.135308 Reply
>>135305
Угу. Лично мне он привил, помимо прочего, любовь к математике. Научил видеть её красоту, что ли. Показал, что математика — это в первую очередь логические доказательства, выводы, а не просто какие-то наборы циферок и буковок, которые нужно зазубрить. С другой стороны, чтоб быть математиком нужны некоторые скиллы, которые у него не примешь. А именно, задачи у него все на понимание и вычисление. Он не учит исследовать самостоятельно, делать доказательства. Вернее учит, но пассививно, собственным примером. Опять же, скилл этот нужен далеко не каждому.
>> No.135309 Reply
>>135308
Это да, я сейчас начинаю понимать, откуда что берется, к примеру. Но не думаю, что буду любить математику прям до ужаса, гуманитарный склад ума не пропьешь, епт.
>> No.135310 Reply
File: algebra-3s-vinber...
Rar, 0.29 KB, 0 files
view
algebra-3s-vinberg-pdf.rar
File: Algebra-[3]-E.B.-...
Pdf, 0.33 KB, 595×842
Algebra-[3]-E.B.-Vinberg.pdf

>>135297
Гм, если ты прямо совсем ничего из этого не знаешь, а экзамен близко, то тебе придётся тяжко. Вот пара книжек, где, как я помню, всё это есть, также помочь может "алгебра" Ван Дер Вардена.
>> No.135312 Reply
>>135310
Ну, теорию я большей частью знаю, но критически не хватает опыта решения задач. Наверное, половину из этих заданий смогу решить, но некоторые не понимаю совсем. Наверное, придется найти добрую душу (ирл), которая поможет со всем этим разобраться.
>> No.135316 Reply
>>135309
Всем, кто к математике теплых чувств не испытвает, а хочет только сдать экзамены и забыть навсегда, я рекоммендую «Математику абитуриенту» Ткачука.
>> No.135320 Reply
Доброчан, помоги. Можешь ли научить на пальцах определять по тексту задачи в теории вероятности, какую из четырёх основных формул комбинаторики использовать?
>> No.135328 Reply
>>135316
Я скорее испытываю к математике некоторое уважение из-за её логичности.
>> No.135333 Reply
File: 1353170205087.png
Png, 0.69 KB, 300×20 - Click the image to expand
edit Find source with google Find source with iqdb
1353170205087.png
>>133509
Что подразумевается, когда говорится о "собственном векторе матрицы A", без упоминания векторного пространства?
>> No.135334 Reply
>>135333
Ну как бы раз есть матрица, векторное пространство уже есть R^n (C^n). Более того, в нём даже выбран базис!
(1, 0, 0, ... , 0), ... , пространство - его линейная оболочка. Потому, что матрица (понимаемая как оператор) это запись оператора в базисе.
>> No.135335 Reply
File: 1421249523120.png
Png, 1.07 KB, 300×20 - Click the image to expand
edit Find source with google Find source with iqdb
1421249523120.png
>>135334
О, спасибо, ясно.
>> No.135385 Reply
Вот сижу я. Дрочу всю эту теорию. Любое доказательство запомнить, понять, повторить - пожалуйста. Тем не менее скилл решения нестандартных задач все хуже и хуже. Куда катится мир..
>> No.135390 Reply
File: hug.png
Png, 672.75 KB, 896×680 - Click the image to expand
edit Find source with google Find source with iqdb
hug.png
>>135385
Saem shit here, bro.
>> No.135407 Reply
>>135385
Действуй по-венгерски: решай задачи.
>> No.135418 Reply
>>135407
Какая мысль! Какое решение!
Тем не менее, других по этому поводу я никогда и не слышал. Хорошо.
>> No.135420 Reply
File: Фото1584.jpg
Jpg, 663.82 KB, 2592×1944 - Click the image to expand
edit Find source with google Find source with iqdb
Фото1584.jpg
Ненавижу это дерьмо. В левой части должно быть х^3*у'', ЧТО Я КАК ВСЕГДА ДЕЛАЮ НЕ ТАК?
>> No.135428 Reply
>>135385
Ну не зря же говорят, что понятая теория - это набор хорошо разобранных и осознанных примеров, а не теоремы и доказательства. Ну то есть ИМХО когда встречешь проблему, ты думаешь не "Ааа, по теореме ^%$#& тут должно быть так", а понимаешь, как оно на самом деле и почему. Теория лежит не дискретным набором теорем, а чем-то вроде общей непрерывной среды. Смысл решения задач не в вспоминании разобранных теорем (что по-моему вполне может снижать способность к решению интересных задач), а именно в мыслительном процессе с учётом понятых теорий, которые работают почти на автомате, как интуиция.
inb4 наркоман.
>> No.135430 Reply
>>135428
Хорошо доказанная теорема - это и есть грамотно разобранный пример.
>> No.135435 Reply
>>135430
Удваиваю.
Я апаю скилл решения задач из-за плохой памяти. Выучил теорему, забыл теорему, сам доказал, забыл, опять доказал.

Решил так письменный экзамен не прикасаясь к задачам месяц, только читая учебник.
>> No.135436 Reply
>>135430
Удваиваю.
Я апаю скилл решения задач из-за плохой памяти. Выучил теорему, забыл теорему, сам доказал, забыл, опять доказал.

Решил так письменный экзамен не прикасаясь к задачам месяц, только читая учебник.
>> No.135437 Reply
>>135420
уже понял
>> No.135439 Reply
Как вообще развивать математическое мышление?
>> No.135440 Reply
>>135428
Какая тут наркомания? Конечно, ты все правильно говоришь, но дело в том, что где я могу понимать на интуиции или так скажем чувствовать теорию, понимать где хватает силы, чтобы что-то выполнялось, где не хватает, я это делаю, но вот шутка, порой бывают ходы, совершенно нелогичные и парадоксальные, которые-таки приводят к результату и они-то мне не даются ну никак. В теореме - запоминаю. В задачке - не решу. И вот как до них-то додумываются - для меня полная загадка. Хочешь конкретики, пожалуйста, я по требованию представлю, но думаю, о чем речь, ясно.
Но замечание все равно очень дельное, не всегда получается ему следовать, хоть и знаешь о нем, спасибо.
>> No.135473 Reply
File: 1421426328528.gif
Gif, 1965.76 KB, 451×257 - Click the image to expand
edit Find source with google Find source with iqdb
1421426328528.gif
>>133509
Анон, как доказать(или где такое доказательство есть) что I максимальный идеал кольца K <=> Факторкольцо K/I - поле?
Желательно простое доказательство с рассмотрением => и <= по очереди.
>> No.135474 Reply
File: l_18.pdf
Pdf, 0.12 KB, 612×792 - Click the image to get file
l_18.pdf
>>135473
Ван дер Варден, алгебра, параграф 16.
Прикреплейтед, теорема 18.8.
>> No.135483 Reply
>> No.135484 Reply
>>135483
Ад.
>> No.135491 Reply
>>135483 покер-фейс не стёр, засранец
>> No.135492 Reply
>>135491
Ты про что?
>> No.135493 Reply
>>135492 trollface.jpg чёрт, уже стал забывать, как называются эти рожи. должны быть абсолютно серьёзное лицо и горящие глаза
>> No.135497 Reply
>>135483
И это происходит в моем вузе... Надеюсь, он хоть ургушный, а не наш.
>> No.135498 Reply
>>135497 вопрос в том, как к этому отнесутся. если с юмором — всё нормально
>> No.135511 Reply
>>135498
Это не юмор же.
>> No.135512 Reply
>>135511 если в университете отнеслись к этому серьёзно, его можно закапывать. ты про это?
>> No.135516 Reply
>>135512
— Да, а что думают преподаватели твоего университета о твоём открытии?
— Прекрасно думают. Все вопросы будут на заседании кафедры

А кого закапывать?
>> No.135522 Reply
>>135516
Универ.
>> No.135523 Reply
> У куба и октаэдра совпадают середины ребер. Найти отношение объема куба к объему октаэдра.
Как же это представить/начертить? Я уже голову сломал.
>> No.135524 Reply
>>135523
Наверное, нужно составить систему уравнений для вершин куба и решить её.
>> No.135527 Reply
>>135524
А чертёж?
>> No.135528 Reply
>>135527
А потом начертить.
>> No.135531 Reply
>>135528
Система уравнений может помочь? Ну ладно. А как её составить? Метод координат применить?
>> No.135532 Reply
>>135531
> Метод координат применить?
Конечно.
>> No.135533 Reply
>>135532
Но ведь для него нужен чертёж.
>> No.135534 Reply
>>135533
Начерти октаэдр, а дальше составь уравнения из условия "совпадают середины ребер", найди вершины куба и соедини их отрезками.
>> No.135542 Reply
File: madskills.png
Png, 4.91 KB, 230×210 - Click the image to expand
edit Find source with google Find source with iqdb
madskills.png
>>135532

Аааа! Не надо координат! Как координаты писать, что куда вписывать-то? Лови мой мэдскиллз с картинкой
>> No.135655 Reply
>>134935
Ты имеешь в виду построение терма в системе? Думаю, если есть правило образования в смысле, как у Бурбаки со знаком тау, то построение есть.
>>134945
Ему на ум пришла ассоциация из-за того, что интуиционизм известен критикой теории множеств и уделяет внимание конструктивности. Ты говоришь о системе, они являются конечными объектами и приемлемы интуиционизмом. В отличие от содержательной интерпретации: интуиционизм не применяет широко теорию множеств.
>>134998
Спутаны понятия вывода и доказательства.
> Я нашёл в первой главе первого тома в одном из Критериев, уже не помню его номер, ошибку перевода.
Пожалуйста, найди снова и покажи мне. Я читал, но не помню такого.
Читал книгу Донченко "Карри Х. — Основания математической логики", которая объявлена переводом первого издания книги Карри "Foundations of Mathematical Logic", и нашёл много ошибок, которых нет во втором издании "Foundations of Mathematical Logic".
>>135010
> Многие вещи действительно гораздо проще воспринимать визуально, если можно так выразиться - во плоти и крови, а не как набор формальных свойств.
Прет, визуал. xD А я обычно считаю, что во плоти и крови — это как набор формальных свойств.
>>135157
Главное — закуп и контроль карты. gg hf
>>135297
> Увы, нету. Могу только задачки семинаров и к.р. скинуть, и надеяться, что что-нибудь такое и будет.
Если есть экзамен, то должна быть программа.
>>135439
Возможно, вас также заинтересует: Что такое математическое мышление? Существует ли математическое мышление? Нахуя развивать математическое мышление?
>> No.135658 Reply
File: ошибочка1.png
Png, 55.09 KB, 715×228
edit Find source with google Find source with iqdb
ошибочка1.png
File: ошибочка2.png
Png, 37.44 KB, 654×146
edit Find source with google Find source with iqdb
ошибочка2.png
File: Начала-Евклида.-К...
Rar, 5.61 KB, 0 files
view
Начала-Евклида.-Книги-I-VI-1950.rar

>>135655
Это небольшая ошибка, почти что опечатка, но неприятная.
> Спутаны понятия вывода и доказательства.
Я против подобной философии. Евклида заформализовали, и весь смысл его книг был утерян, теперь учебники геометрии читать нельзя, от них тошнит. Смысл доказательства в том, чтобы делать неочевидное очевидным, а не в том, чтобы писать строки символов по бюрократическим правилам.
Кстати о Евклиде. Ты читал его первую книгу? По-моему, формалист не способен её прочитать.
>> No.135702 Reply
File: DaL.jpg
Jpg, 124.32 KB, 706×1024 - Click the image to expand
edit Find source with google Find source with iqdb
DaL.jpg
Дорогие аноны, подскажите с задачей? С меня как обычно.

Стоимость проезда по платной трассе длиной в 1000 км из города Ближний в город Дальний составляет 10 рублей за каждый километр первой четверти пути и уменьшается вдвое каждую последующую четверть. Причём качество дорожного покрытия и режим движения по трассе позволяют снизить расход бензина до 10
литров на 100 км. Если ехать обычной дорогой, то расстояние и расход бензина увеличиваются в полтора раза. Но после первой трети пути обычная дорога имеет съезд на платную трассу, оказываясь в её середине. Какую наименьшую сумму заплатит Аристарх Луков-Арбалетов, поехавший из Ближнего на маминой машине к друзьям в Дальнем, если из-за снижения мировых цен на нефть бензин подорожал до европейского уровня и стоит 100 рублей за литр?
>> No.135704 Reply
У меня вышло 11 718 750 по платной, 1 500 000 по бесплатной и 1 296 875 по совмещённой.
>>135702-кун.
>> No.135705 Reply
File: ss+(2015-01-21+at+05.25.56).png
Png, 6.27 KB, 583×93 - Click the image to expand
edit Find source with google Find source with iqdb
ss+(2015-01-21+at+05.25.56).png
Тут есть кто-нибудь? Заранее благодарю.
>> No.135708 Reply
>>135702
Ты умножить/поделить не можешь что ли? В каком классе? Напомню, что это /u/ и кафедра математики, а не кружок арифметики.

250х(10+5+2,5+1,25) по платной за проезд +100х100 за бензин. (100 литров на 1000 километров)
14687,5

100х1,5х100х1,5=22500 за бензин на бесплатной

С совмещённой трассой, полагаю, считать нужно, что при переходе на неё стартовая цена 10 рублей, а далее скидка начинается. Потому что
> уменьшается вдвое каждую последующую четверть
> четверть
эта четверть может быть трактована ещё двумя способами: четверть запланированного пути или на каждой четверти трассы своя фиксированная цена.


Допустим мы проезжаем треть по обычной, тратим треть суммы: 22500/3=7500
Дальше 2/4 платной трассы это 250х(10+5) за проезд + 50х100 за бензин = 8750
Итого 16250 по комбинированной.
> Аристарх Луков-Арбалетов, поехавший
доверенность на мамкину машину не дали, значит это всё лишь его больная фантазия, никуда он не ездил.

>>135704
каким образом у тебя миллионы получились?
1000 километров х 10 литров х 100 рублей штоле?
Внимательно к размерности данных:
1000(км) х 10(л)/100(км) х 100(р/л)
сокращаем размерности: кмлр/км/л = р.

1000(км) х 10(л) х 100(р)=1000000 километро-литро-рублей.
>> No.135711 Reply
>>135658
> ошибочка1.png
Спасибо. Возможно, попадалась.
> Смысл доказательства в том, чтобы делать неочевидное очевидным, а не в том, чтобы писать строки символов по бюрократическим правилам.
Согласен, дело обстоит так с доказательствами и противоположным образом с выводами. Можешь рассматривать вывод как часть доказательства эпиутверждения.
К: композиция
>> No.135713 Reply
>>135708
Спасибо, что всё пояснил по хардкору, но... ответ "14687,5" не верен. Мне нужно вбивать ответ в специальный сайт препода.
>> No.135715 Reply
Может кто-нибудь, пожалуйста, помочь с математикой? Первый курс.
>> No.135716 Reply
>>135715
Может. Какие вопросы?
>> No.135718 Reply
>>135716
Я в принципе не догоняю. Возможно, только на совсем элементарных примерах для даунов пойму. И вопросов слишком много.
>> No.135723 Reply
>>135718
Найди какую-нибудь методичку по теме.
>> No.135727 Reply
>>135713
> но... ответ "14687,5" не верен.
Логика задачи подробно расписана. Проверь и убедись. Мне насрать на то, что у препода может быть неправильный ответ. Я тоже учился в школе, и качество некоторых задачников было настолько низким, что 5-10% ответов были или неверными, или неграмотно представлены. Например вместо 2/3 — 0,66, хотя при округлении 0,67, а так же существует запись 0,(6) — значит, что шесть в периоде.

Вас там не учат что ли грамотно расписывать «Дано», делать чертёж/таблицу/уравнения, расписывать решение, а потом делать сверку? Значит учись сам. Нарушение или халатное отношение к методике — одна из самых частых причин ошибок.

>>135718
Ты в принципе ни за что не брался и не собираешься, раз даже вопросов нет. «Пусть меня научат», да?
>> No.135728 Reply
>>135713
14687,5 or 14687.5 or 14688 or 14687 пробовал?
>> No.135729 Reply
File: Calculus-made-easy.-Thompson.pdf
Pdf, 11.39 KB, 407×500 - Click the image to get file
Calculus-made-easy.-Thompson.pdf
>>135715 мог-мог-мог... помог
>> No.135736 Reply
>>135727
>>135728
Ответ 13 тысяч с чем-то. Один человек с отцом-математиком разобрались за 40 минут.
>> No.135740 Reply
>>135736
Каким образом?
Их решения и логики я не вижу. Своё перепроверил, нигде вроде не ошибся. Зови аццов, пусть разъяснят чо пачом
>> No.135742 Reply
File: nzxpz03NsAg.jpg
Jpg, 144.01 KB, 1088×816 - Click the image to expand
edit Find source with google Find source with iqdb
nzxpz03NsAg.jpg
Добрый вечер, чане.
Сейчас у меня полночь, через 15 часов у меня пересдача экзамена по математике.
Школьная математика у меня особых трудностей не вызывала, но в вузе как-то не очень удается.
Есть один из полусотни вариантов. Какие-то остаточные знания у меня все равно в голове присутствуют, да и память неплохая, но совершать много мелких вычислений мне довольно трудно, потому что я легко теряю знаки, либо просто цифры вылетают из выражений.
   К чему мне легче всего подготовиться? Ночью я спать буду, но днем с утра буду готовиться, думаю. Набрать мне нужно минимум 17 баллов из 40
Задания пикрелейтед. Разбалловка ниже.

1. Ответить на вопрос. (2 балла).
2. Вывести уравнение, формулу. (5 баллов)
3. Сформулировать и доказать теорему ... (5 баллов)
4. Решить матричное уравнение ... (8 баллов).
5. Составить каноническое уравнение эллипса (гиперболы), если известно, что ... . Сделать чертёж. (4 балла)
6. Какую линию определяет уравнение ... ? Найти её эксцентриситет. Сделать чертёж. (6 баллов)
7. Найти пределы: а) ... ; б) ... ; в) ... . (6 баллов)
8. Исследовать функцию ... на непрерывность в точках ... и ... . (4 балла)
>> No.135744 Reply
>>135742
1-просто
2-написать теория просто, оформить через формулы достаточно сложно.
3-я их заучивал.
4-элементарно.
5-просто, но уже не помню
6-это вообще школа, разве нет?
7-всегда любил пределы, очень легко давались.
8-непрерывности тоже любил тоже легко всё.

Но это моё личное восприятие.
я бы взял по убыванию 8-6-7-4, первое - как я понял просто совсем, а остальное уже подчитал под конец не сильно делая на него упор, но дело твоё, не знаю что тебе легче даётся
>> No.135747 Reply
>>135736
13437.5
>>135708
> Дальше 2/4 платной трассы это 250х(10+5) за проезд
Нет, верно 2.5х250+1.25х250=937.5
Он начинает платную трассу с середины, поэтому скидка начнётся с 2.5 рублей, а не с 10.
>> No.135759 Reply
>>135747
Кажется, вы имеем дело с кретинской формулировкой задачи. Ибо можно истолковать систему оплаты как угодно, получить при этому кучу разных ответов.
>> No.135760 Reply
>>135747
> Он начинает платную трассу с середины, поэтому скидка начнётся с 2.5 рублей, а не с 10.
Про невнятность трактовки четвертей я упоминал. Но сфига ли по логике сразу четырёхкратная скидка тому, кто только въехал на трассу + стимулировать въезд тех, кто будет мешать водителям, въехавшим с начала платной трассы и заплатившим гораздо дороже?

В условии сказано «10 рублей за каждый километр первой четверти пути». То есть, если строго следовать этому условию, то нужно весь путь, а именно вторую половину платной трассы при комбинированном проезде, делить на четыре части и назначать цену, начиная с 10 рублей.

При таком варианте 125*(10+5+2,5+1,25)=2343,75 за проезд второй половины платной трассы.

Короче не понятно, намеренная ли путаница в условии и каковы точные условия скидки.
>> No.135762 Reply
>>135759
Мы имеем дело с составителем, считающим, что его логика очевидна и у всех остальных ровно такая же. Особенно «любимы» мною за подобное составители «детских элементарных задач», считающие, что в простой задаче при составлении ошибиться нереально (что и является первой грубой ошибкой).
>> No.135795 Reply
Какие разделы математики нужно изучать будущим химикам-инженерам и биоинженерам? Какие хорошие специализированные учебники на русском/английском для них есть?
>> No.135796 Reply
File: index.jpg
Jpg, 6.24 KB, 225×225 - Click the image to expand
edit Find source with google Find source with iqdb
index.jpg
>>135795
Вангую что по биохимии тебе книжки читать надо!
>> No.135797 Reply
>>135795
Никакие. Тебе нужны только несколько формул.
>> No.135798 Reply
>>135795
Численные методы всякие, для биоинженеров - программирование на языке R, например.
>> No.135799 Reply
File: sample_50c030700fd761958050ea5df045f140.jpg
Jpg, 351.58 KB, 850×588 - Click the image to expand
edit Find source with google Find source with iqdb
sample_50c030700fd761958050ea5df045f140.jpg
>>135795
Эхэхэ, не в тот ты тред вопрос задал, тут же люди врядли знают, что нужно биохимикам. От балды видимо будет полезна хотябы чуть гидродинамика, мсс, самое кошерное, на ум приходит книга Арнольда топологические методы гидродинамики, но там скорее "дополительные главы для математиков"), хотя там много красивых вещей, которые могут пригодится, диффуры наверняка, опять же посмотри его книжки по диффурам, если хочется понять-почувствовать красивые вещи и есть время.

А вообще определись, какие разделы и насколько глубоко тебе нужны. Вообще будет полезно в любом случае человеку технической специальности поднять общий математический уровень.
>> No.135800 Reply
>>135799
Нет, это всё не нужно прикладникам. Им нужны только три с половиной формулы и объяснение, какие числа в них подставлять.
>> No.135802 Reply
>>135800
Может быть ты и прав, но это в зависимости от уклона, чистому прикладнику пожалуй да, и то я бы посомневался, мне кажется, что человек с более высокой эрудицией в смежных областях может добиться большего в своей. Ну то есть универсальные учёные прошлого куда-то делись, и не мудрено, но всё же по-моему стоит хотя бы брать их подход насколько это возможно.

Хотя это немного грустно.
>> No.135805 Reply
>>135800
Ты не прав, я вон автоматизатор химпроизводств, собственно и сами процессы мы изучаем, так у нас есть такое понятие как матмодель и управление процессом, вполне вероятно, что там им тоже прийдётся таким заниматься, а там и дифуры и зет-преобразования, преобразования фурье собственно это я применял на практике и матрицы, в общем весь курс вышки нужен, а используется всё относительно поставленной задачи.
Это для химиков-ингенеров.
Для биологов не ебу.
выучи два пакета: маткад и матлаб, прослушай курс лекций по вышке чтоб занать для чего что применять да и больше ничего не надо, кампутер всё делает за тебя
>> No.135807 Reply
>>135805
Видишь ли. Вот то, что ты перечислил, - преобразование Фурье, матрицы, - это такая элементарщина, что говорить о них как о чём-то значимом, я думаю, не стоит. По идее, их надо в школе изучать.
>> No.135808 Reply
>>134998
(>>134538)
> Задача 5
> Докажите, что для произвольных отображений f, g, h выполняется следующее: h ∘ (g ∘ f) = (h ∘ g) ∘ f
По определению композиции отображений f:X → Y, g:Y → Z, f ∘ g = h:Y → Z. Иными словами, если f = {f(x) ∈ Y ∣ x ∈ X}, g = {g(f(x)) ∈ Z ∣ f(x) ∈ Y} то очевидно, что область определения отображения g должна совпадать с образом множества f[X] = Y.
Теперь пусть f:X → Y, g:Y → Z, h:Z → W
h ∘ (g ∘ f) = h ∘ ((g ∘ f)(x)) = h ∘ (g(f(x))) = h(g(f(x)));
(h ∘ g) ∘ f = (h ∘ g)(f(x)) = h(g(f(x)));
Очевидно, что h ∘ (g ∘ f) = (h ∘ g) ∘ f.
> Задача 6
> нарисуйте картинки для а) g ∘ f; б) h ∘ g; в) f ∘ h ∘ g; г) g ∘ h ∘ f:
а) g ∘ f = {<3, 3>, <1, 137>, <2, 137>, <5, 7>};
б) h ∘ g = {<0, 1>, <1, 2>, <2, 3>};
в) f ∘ h ∘ g = {<1, 2>, <2, 2>, <3, 1>, <5, 3>};
г) g ∘ h ∘ f = {<0, 1>, <1, 1>, <2, 0>};
Тут что-то нечисто. Композиция не коммутативна? Кажется да. Но даже в первом примере мы в таком случае уже не можем получить композицию. Образ g не является областью определения f. Что же тогда? Толковать применять последовательное отображение справа налево? Тогда отваливается г). Не понимат.
> Задача 7
> Про каждое из отображений на рисунке выясните, является ли оно биективным:
а) не является (f(5) = f(11) = 4); б) является, отображение задаёт взаимно-однозначное соответствие между множествами {3, 5, 7, 11} и f[{3, 5, 7, 11}]; в) не является |{3, 5, 7}| ≠ |f[{3, 5, 7}]|, или иными словами, ∃y ∈ Y, f⁻¹(y) = ∅; г) не является, то же самое, что для в).
> Задача 8
> Нарисуйте все биективные отображения а) из множества {1, 2} в множество {3, 4, 5, 6}; б) из множества {1, 2, 3} в множество {4, 5, 6}:
а) request cannot be processed. Мощности множеств не совпадают, значит отображение не может быть взаимнооднозначным.
б) {<1, 4>, <2, 5>, <3, 6>} и ещё 8 вариантов.
> Задача 9
> Пусть f:X → Y, g:Y → Z. Верно ли, что если f и g биективны, то и g ∘ f биективно?
Биективность отображение f:X → Y означает, что ∀y ∈ Y, ∃x ∈ X, f(x) = y, т.е. f⁻¹(y) ≠ ∅ и f(x) ≠ f(x'). Значит g⁻¹ ∘ f⁻¹ = (g ∘ f)⁻¹ ≠ ∅. Если (g ∘ f)⁻¹ = ∅, значит область определения g и f не существуют. Функции не определены. Нельзя задать композицию неопределённых фукнций. Но это лишь один из криетриев. Второй: взаимная однозначность. Так как оба отображения взятые по отдельности взаимооднозначные, то их композиция также даст взаимооднозначное отображение, иными словами (g ∘ f)(x) ≠ (g ∘ f)(x') и |(g ∘ f)[X]| = |(g ∘ f)⁻¹[X]|.

Альзо, два момента меня несколько беспокоят. Я уже упоминал. Последовательное применение отображений: так какова же всё-таки эта последовательность? И коммутативность композиции.

Извиняюсь. Наконец выбрался запостить хоть что-то. К сожалению, дела идут не очень и настали плохие времена, так что дальше возможно будет ещё хуже. Но постараюсь превознемочь.
>> No.135824 Reply
Есть 99 вопросов в экзамене. Раньше порядок был таков: 1-34-99, 2-35-98 и так далее. Вот несколько билетов актуальных: 23-66-40. 25-38-68. 11-54-97. 14-57-39. В чём новая логика?
>> No.135825 Reply
>>135824
Подобные вопросы нужно решать экспериментально.
enwiki://Peter_Cathcart_Wason#Wason_and_the_2-4-6_Task

Поскольку у нас нет доступа к экспериментам, ответ на твой вопрос мы получить не можем.
>> No.135826 Reply
>>135825
Сие печально.
Ещё один билет! 11-55-98
>> No.135830 Reply
>>135826
+ ещё один 30 73 78
>> No.135836 Reply
>>135830
Судя по 11-54-97 и 11-55-98 никакой схемы нет, тупо рандомом набросали.
>> No.135882 Reply
>>135802
> Ну то есть универсальные учёные прошлого куда-то делись, и не мудрено
А правда, куда они делись? Inb4: умерли.
Математика стала такой сложной, что широким специалистом стать почти невозможно в возрасте всяких там Галуа?
>> No.135910 Reply
>>135882
Ну примерно, дело не только и не столько в математике. В конце XIX, а особенно в XX веке число учёных вообще стало расти по экспоненте, в итоге науки очень сильно продвинулись и в ширь и вглубь. Теперь человеку чисто физически не хватает времени, чтобы познать весь огромный открытый объём знаний. Возможно есть и другие причины, например изменилось понятие учёного. Я имею в виду то, что под учёным зачастую понимается узкий специалист и люди следуют этому тренду тоже. Думаю, что причин много на самом деле.
>> No.135911 Reply
>>135808
5. Ок.
6.
> Композиция не коммутативна?
Да. Это была просто проверка на понимание.

7. Ок.
8. А точно ли восемь? Единица должна отобразиться либо в 4, либо в 5, либо в 6, то есть в один из трёх элементов. Выберем какой-нибудь вариант. Двойка тогда может отобразиться только в два элемента. Тройка - только в один элемент.
9.
> Биективность отображения f:X → Y означает, что ∀y ∈ Y, ∃x ∈ X, f(x) = y,
Контрпример - 7а. Это ты сюръективность описал. Для биективности нужно требовать единственность икса, это обозначается почти как ты написал, только с восклицательным значком после квантора существования: ∃!, "существует единственное". Хотя сюръективность следует из биективности, это верно.
> g⁻¹ ∘ f⁻¹ = (g ∘ f)⁻¹
Сначала надеваем рубашку, потом надеваем пиджак. А снимаем в обратной последовательности, сначала пиджак, потом рубашку.
> Так как оба отображения взятые по отдельности взаимооднозначные, то их композиция также даст взаимооднозначное отображение
Ок.
> так какова же всё-таки эта последовательность?
Вообще, это спорный вопрос. Одни авторы определяют (g∘f)(x) как g(f(x)), другие как f(g(x)). Спор любителей левоассоциативности с любителями правоассоциативности напоминает свифтовский спор между остроконечниками и тупоконечниками.
ruwiki://Очерёдность_операций

Почти стандартом стал вариант, когда (g∘f)(x) определяется как g(f(x)). (h∘g∘f)(x) определяется как h(g(f(x))).
Над (g∘f)⁻¹ подумай получше. Я не зря про пиджак сказал.
> Наконец выбрался запостить хоть что-то.
Это хорошо, я уже волноваться начал.
> настали плохие времена
Добра тебе, няша.
>>135882
Несколько сот лет назад математик мог изучить доказательства всех существующих математических теорем. Сейчас книг по математике стало так много, что один человек не в силах их прочитать.
>> No.135917 Reply
>>135882
>>135910
>>135911
А кого мы вообще называем "широким специалистом"? Серр, например, успешно занимался и алгебраической топологией, и алгебраической геометрией, и теорией групп, и теорией чисел. Делает ли это его широким специалистом или мы отказываем ему в этом звании на основании того, что все эти области близки к алгебре, а, значит, Серр по сути алгебраист?
>> No.135918 Reply
>>135917
Того математика, для которого нельзя указать область, в которой он не оставил работ.
>> No.135924 Reply
>>135918
Тогда меня очень интересует а)пример такого математика; б)список областей.
Ибо если брать все области математики, существующие ныне, то окажется, что у широких специалистов прошлого, как правило, нет работ, скажем, по геометрической теории групп. Значит, ты неявно имеешь в виду довольно узкий список "основных" областей математики.
>> No.135925 Reply
Посоветуйте книгу по теории вероятностей для новичка.
>> No.135926 Reply
>>135924
*современную ему область
Концевич. По-моему, он логикой не интересуется. inb4: теорию категорий от логики всё-таки надо отделить.
Список современных нам областей более-менее совпадает со списком на Архиве.
>> No.135943 Reply
Что гениального в числе Пи?
>> No.135950 Reply
>>135925
Письменный, конспект лекций по теорверу.
>> No.135966 Reply
Структуры и алгоритмы обработки данных.

Как вычислять R^-1 я так и не нагуглил, но в голове осталсь инфа о том, что надо поменять местами первый и второй элемент множества, т.е. будет R^-1 = {y, x}. Окей, тогда S^-1 = {b, a}. Но как композицию-то считать? Ведь общего соответствия нет.
>> No.135967 Reply
Алсо, посоветуйте учебник матана для гуманитария, который проспал все уроки математики начиная с шестого класса.
>> No.135968 Reply
File: Фото0091.jpg
Jpg, 81.48 KB, 1988×320 - Click the image to expand
edit Find source with google Find source with iqdb
Фото0091.jpg
>>135966
Пик отклеился, все мозги уже этими заданиями запрессовал себе, спать охота.
>> No.135969 Reply
>>135966
> Как вычислять R^-1 я так и не нагуглил
http://neerc.ifmo.ru/wiki/index.php?title=Композиция_отношений
Смотри строчку
> Отношение R^{-1} \subseteq B\times A называют обратным (англ. inverse relation) для отношения...
>>135967
> один учебник
> начиная с шестого класса
Смеешься?
>> No.136012 Reply
File: 14094846212510.jpg
Jpg, 262.77 KB, 1024×1024 - Click the image to expand
edit Find source with google Find source with iqdb
14094846212510.jpg
>>135969
> http://neerc.ifmo.ru/wiki/index.php?title=Композиция_отношений
За ссылку сябки)))), никак что-то подобное найти не мог.
> Смеешься?
Понимаю, что в один учебник тут не уложишься, но хотя бы цикл тогда.
>> No.136015 Reply
>>136012
Пожалуйста.
Тогда прочитай внимательно первые четыре поста из треда >>124265 и тексты по ссылкам из них, я его специально для таких случаев написал. Там много текста, но ты назвался гуманитарием, поэтому, думаю, осилишь.
>> No.136018 Reply
Как вообще вы определяете: гуманитарий или технарь?
>> No.136024 Reply
> Сколькими способами можно составить трехцветный полосатый флаг (три горизонтальные полосы), если имеется материя из 5 цветов?
Для цветов: 5!/(5-3)!=60, что и есть ответ, тогда как решить для четырехполосного флага в 3 цвета?
>> No.136026 Reply
>>136024
А как ты себе это представляешь? Ты отвергаешь принцип Дирихле или у тебя допустимы повторения?
>> No.136045 Reply
>>136026
> принцип Дирихле
Что это? Допустим, пустой цвет, это бред, да?
>> No.136047 Reply
>>136045
> Если число клеток больше, чем число кроликов, то как минимум одна клетка пуста.
ruwiki://Принцип_Дирихле_(комбинаторика)
>> No.136048 Reply
>>136047
How to solve it есть на русском и, если есть, то как называется?
>> No.136052 Reply
File: Снимок.PNG
Png, 9.96 KB, 665×237 - Click the image to expand
edit Find source with google Find source with iqdb
Снимок.PNG
А что, если в самом начале представить второе уравнение, как "А+А+А+А+B+B+C+C+C=4" и воспользоваться первым равенством? Тогда получится, что "2A+C=4". Корни же тут - -1;-5;6. И полученное мною равенство не выполняется. Вопрос: что я делаю не так?
>> No.136053 Reply
File: 140242984889407561.jpg
Jpg, 44.46 KB, 800×600 - Click the image to expand
edit Find source with google Find source with iqdb
140242984889407561.jpg
к слову о теорвере. мужики спасайте, горю!
У охотника 4 патрона. Он стреляет по зайцу, пока не попадет или пока не кончатся патроны. Вероятность попадания примерно 0.25.
Составьте закон распределения. Найдите математическое ожидание количества выстрелов и дисперсию
решение нагуглил, а оно валидное? меня смущает х=0
>> No.136058 Reply
Доброанон, выручи идиота, пожалуйста.
Нужно понять вектора. Сам я их осознаю очень смутно, и словами обьяснить что это такое не смогу. Так же очень надо врубиться, как решать примеры вида - x=a+jb (где x,a,b - векторные величины).
В связи с этим желанием реквестирую учебников по математике (я даже не знаю, к какому разделу математики это относится). Где про это читать?
Обьяснить не прошу, дайте только название книжки, школьный учебник подойдет.
>> No.136059 Reply
>>136058
Кострикин, "Введение в алгебру".
Вектор - это элемент векторного пространства над каким-то полем (элементы поля называются скалярами).
Векторным пространством называется множество, над которым можно совершать действия двух типов.
Во-первых, два его элемента можно складывать.
Во-вторых, любой его элемент можно умножать на скаляр.
Причём эти действия обладают некоторыми свойствами.
Всего этих свойств восемь.
ruwiki://Векторное_пространство

Ты знаешь, что такое определение математического объекта? Мы берём какие-то свойства и объединяем их в один какой-то мыслимый объект, которому затем даём имя. И любой объект, среди свойств которого есть избранные нами свойства, мы потом называем этим именем.
Когда мы начинаем работу с каким-то объектом, мы отказываемся от всяких сведений нём, кроме тех, которые явно указаны в его определении. Мы работаем с векторами только с помощью тех восьми свойств, которые указаны в определении. Никакие "направленные отрезки" и прочее мы не задействуем. Только восемь свойств, восемь определяющих аксиом.
>> No.136060 Reply
>>136052
> Корни же тут - -1;-5;6. И полученное мною равенство не выполняется.
Выполняется же. 2A + 0B + 1C = 2(-1) + 0(-5) + 6 = -2 + 0 + 6 = 4.
>> No.136061 Reply
>>136059
Спасибо, нашел, скачал, буду осознавать.
Определение математического объекта если и знал, то забыл.
>> No.136064 Reply
>>136060
Точно. Ну и криворукий же я. Перепроверил сотню раз и всё равно почему-то не то получалось.
>> No.136084 Reply
File: прогрессия.PNG
Png, 1.57 KB, 137×30 - Click the image to expand
edit Find source with google Find source with iqdb
прогрессия.PNG
Почему это свойство выполняется только в арифметической прогрессии?
>> No.136085 Reply
>>136084
Из определения арифметической прогрессии.
>> No.136093 Reply
File: уця.PNG
Png, 81.66 KB, 810×578 - Click the image to expand
edit Find source with google Find source with iqdb
уця.PNG
Помогите перевести. И еще, разве (DUH)^с разве не равно АUB? "А" здесь куда-то подевалось и только В осталось :с
>> No.136095 Reply
>>136093
Возьмите карту из обычной колоды в 52 карты. The sample space S - множество всех 52 карт (то есть имеются 52 шара, один для каджой карты).
Рассмотрим следующие четыре события.
A. Карта - туз
B. Карта чёрной масти.
D. Карта - буби.
H. Карта - червы.
Как множество, H состоит из 13 карт: {туз червей, двойка червей, ... , король червей).
Мы можем определить различные другие события с помощью A,B,D,H. Например, A в пересечении с H есть событие, когда карта есть туз червей, A в пересечении с B есть событие {туз пик, туз треф}, а событие A в объединении с D в объединении с H есть событие, когда карта красной масти, либо туз. Заметим, что дополнение к объединению D и H есть пересечение дополнения к D и дополнения к H, что равно B, поэтому B может быть выражено через D и H. С другой стороны, событие, что карта - пики, не может быть выражено через события A,B,D,H, поскольку ни одно из них не детализировано достаточно, чтобы отличить пики от треф.
>> No.136125 Reply
Как не потерять корни при решении уравнений?
>> No.136126 Reply
>> No.136127 Reply
File: tarot_fool.jpg
Jpg, 89.47 KB, 267×449 - Click the image to expand
edit Find source with google Find source with iqdb
tarot_fool.jpg
Насколько реально сейчас доморощенному анону в сириуз математику? И чем там занимаются?
Когда читаю некоторые форумы тематические - охуеваю. С литературы математической охуеваю ещё больше - понятно чуть больше, чем кусочек нихуя; причем не то, чтобы материал трудный - очень много времени тратишь на переваривание формата и понимаешь, что можно было бы переписать, ориентируясь на читателя. короче
нить иди, короче
>> No.136130 Reply
>>133509
Кто делал ОП-пик? Годнотень, но почему кисть такая поехавшая?
>> No.136138 Reply
> очень важно знать свойство непрерывности вещественных чисел, описанное Дедекиндом
http://gen.lib.rus.ec/book/index.php?md5=A3E47963688DDAAC6B576FA97F9D5293 Это оно? Какой-то объем маленький, ну ладно
>> No.136141 Reply
Давайте считать количество всевозможных судоку-табличек. Руками, компьютер нельзя трогать.
>> No.136142 Reply
>> No.136143 Reply
>>136141
Без учета ограничений на квадраты 3х3 таких таблиц будет 9!х8!х...х1! Как тут остальные варианты, я понятия не имею.
>> No.136144 Reply
>>136138
"У любого ограниченного множества вещественных чисел существует точная грань".
Формулировка простая, но само по себе свойство удивительное. Удивительно оно тем, что оно не зависит от способа формирования указанного множества вещественных чисел. Каким бы головоломным образом мы ни описали бы множество M ⊂ ℝ, если M ограничено, то у него будет точная грань.
Например, скажем, что M - все такие вещественные числа, квадрат которых меньше или равен 2. Тогда у M будет точная верхняя грань, которую мы тут же обозначим как √2 и таким образом получим операцию извлечения квадратного корня из произвольного натурального числа. И таким же образом мы все привычные операции на R определяем.
Мы можем определять M гораздо более хитрыми путями, и всё равно это свойство будет иметь место. Например, пусть у нас есть отрезок [a;b], пусть x - это такие точки этого отрезка, что отрезок [a;x] обладает свойством P. Если множество точек x ограничено сверху, то у него есть точная грань. Значит, в [a;b] найдётся подотрезок наибольшей длины, который обладает свойством P вместе с каждым своим подотрезком.
У других привычных чисел - целых, рациональных - такого полезного свойства нет.
>> No.136145 Reply
File: 416098_original-(1).jpg
Jpg, 18.96 KB, 521×122 - Click the image to expand
edit Find source with google Find source with iqdb
416098_original-(1).jpg
>>136127
> Насколько реально сейчас доморощенному анону в сириуз математику?
Пикрелейтед имеет место - всё получится.
> И чем там занимаются?
http://arxiv.org/find/math/1/au:+Mochizuki_T/0/1/0/all/0/1
>> No.136146 Reply
>>136144
Спасибо, это откуда ты взял.
Я уже начал читать его книжку, вот оттуда:
> Прямая L бесконечно более богата точками, чем область R рациональных числе числами.
Не понимаю, числа только представлены в виде точек для наглядности, разве нет? R и L равнозначны по сути, между двумя точками, соотвествующими цифрам 2 и 3 будет бесконечно много точек как бесконечно много чисел есть между 2 и 3.
>> No.136147 Reply
>>136146
> R и L равнозначны по сути
Нет. Мощность множества рациональных чисел меньше мощности множества вещественных чисел. Бесконечности бывают разные. Рациональных чисел столько же, сколько натуральных. Вещественных чисел больше.
>> No.136148 Reply
>>136147
> Мощность множества
Отличное свойство. Интересно, почему не назвали «густотой», «жирностью» или «калорийностью»?
Даже заглянул, что за вещественные числа. У нас их действительными звали.

Но почему так? Ведь дробей тоже сколько угодно можно придумать и любое вещественное число представить дробью, но не любую дробь можно отразить конечным вещественным числом.
>> No.136150 Reply
>>136148
> и любое вещественное число представить дробью
Обыкновенной? Нет. √2 не представимо в виде p/q.
>> No.136151 Reply
Про все эти непрерывности и бесконечно малые/большие есть какие-то книги на английском? А то я не понимаю книгу Рудина, может если такие прочитаю, то пойму
>> No.136152 Reply
>>136150
моя логика даёт сбой, ибо корень из двух литров никак не хочет считаться «вещественным» и по ощущениям ближе к комплексному представлению числа, нежели к простому. Хотя один икс 0,1 то же самое, что 1/10 буквально и где граница сложности, понять нельзя.
>> No.136153 Reply
>>136152
А, опять ты, Золотце.
>> No.136155 Reply
> 5 авторов должны написать книгу из 14 глав. Два автора напишут по 2 главы книги, два других - по 3 главы, и последний - 4 главы книги. Сколькими способами можно распределить главы между ними?
Порядок книг не важен, тогда два автора пишут 4 главы книги всего, которых можно "вынуть" из 14 глав 14!/4!*10! способами. Для двух других 6 глав 10!/6!4! (4 главы уже распределены), для последнего 4 главы остается. Полученные результаты перемножаю, ответ не сходится. Ну и что не так?
>> No.136160 Reply
>>136155
> 14!/4!*10! способа
Почему? ruwiki://Размещение - это не ruwiki://сочетание
>> No.136165 Reply
>>136151
А зачем тебе именно на английском?
>> No.136166 Reply
>>136160
Порядок выбора глав не важен, поэтому сочетание я выбрал.
>>136165
Потому что на русском нормальных книг нет.
>> No.136168 Reply
>>136166
> Потому что на русском нормальных книг нет.
Почему же, у Зорича или у Ильина и Позняка вполне годные книги.
А вообще, можно просто зайти на libgen, вбить туда «real analysis», и найти там, к примеру (рандом), http://libgen.org/book/index.php?md5=FD716FB79496263C2C7167ED74C7DC11 или http://libgen.org/book/index.php?md5=E5914478D2189F4A82EB21A3CDA6E8D6. Ну или http://libgen.org/book/index.php?md5=14ce15a2729f7d943351f8c692879a24 (если вбить «mathematical analysis»).
>> No.136169 Reply
>>136168
Так это всё книги об нескольких разделах кратко, а мне нужная толстая книжка об
> Про все эти непрерывности и бесконечно малые/большие
Не знаю, что "вбивать", чтобы их искать.
>> No.136170 Reply
>>136160
Вот если было бы:
> Два автора напишут 1, 2, 3, 4 главы книги, два других - 5, 6, 7, 8, 9, 10 главы, последний - 11, 12, 13, 14 главы.
Тогда было бы размещение, потому что есть чёткий порядок.
>> No.136179 Reply
File: ываывыаыававыав.PNG
Png, 115.01 KB, 801×824 - Click the image to expand
edit Find source with google Find source with iqdb
ываывыаыававыав.PNG
>>136168
Ага, очень. Я вот не могу вывести 5) из 6), а он сичтает, что это очевидно. И что это за поправочный вектор такой, ясно разве что интуитивно, как с ним работать - непонятно.
>> No.136181 Reply
«Задаётся без помощи метрики», а как же, по-моему она как раз с помощью метрики и задаётся.
>> No.136182 Reply
File: image.jpg
Jpg, 354.29 KB, 1531×870 - Click the image to expand
edit Find source with google Find source with iqdb
image.jpg
>> No.136184 Reply
>>136181
Да, но косвенно. Хоть метрика R² и используется для задания топологии, для самого пространства треугольников метрика не определена.
>> No.136185 Reply
>>136184
Фактически и на треугольниках задаётся — это наибольшее из расстояний между вершинами (при нумерации, в которой оно (это наибольшее) наименьшее, лол).
>> No.136193 Reply
>>136185
А неравенство треугольника (каламбур же) выполняется?
>> No.136194 Reply
File: 123.PNG
Png, 146.09 KB, 881×904 - Click the image to expand
edit Find source with google Find source with iqdb
123.PNG
>>136179
Что-то никто не пояснил. Жалко
Вот еще пикча. Это ведь ошибка, да? Не особо понял этот абзац, почему 5*6?
>> No.136196 Reply
>>136193
Конечно.
>> No.136197 Reply
File: высота.PNG
Png, 67.93 KB, 476×884 - Click the image to expand
edit Find source with google Find source with iqdb
высота.PNG
Что это за чудеса? Как выводятся все эти формулы? Что-то у меня не получается.
>> No.136205 Reply
File: NewCanvas.jpg
Jpg, 113.12 KB, 1151×896 - Click the image to expand
edit Find source with google Find source with iqdb
NewCanvas.jpg
>>136197
Пробуй, выводи, переписав все закономерности в большом треугольнике и двух разделённых.
Кто кому катетом приходится, кто кому гипотинузой, понятие косинуса и синуса (cos[a]=c2/b), теорему Пифагора, теоремы синусов и косинусов. Построение прямоугольника по треугольнику.

Попробовал размять мозг и таки нашёл подходящее преобразование между с, с1, с2
>> No.136206 Reply
>>136205
п.с. вообще я мимохудожник, но сегодня что-то на матан тянет. Вздохнул с облегчением, когда решил: до уровня школьника дотягиваю всё ещё.
>> No.136208 Reply
>>136205
п.п.с. заметил, что вторая формула из первой выводится стремительно, обратной заменой a и b на c*cos. Думаю, остальные так же, но уже лень ковырять.
>> No.136235 Reply
File: Д1.bmp
Bmp, 337.05 KB, 1280×1024 - Click the image to expand
edit Find source with google Find source with iqdb
Д1.bmp
> "Как быстро меняется y при изменении х"
Мне не нравится вот это. Почему именно отношение? Допустим, пусть а=3, h=2
(f(3+2)-f(3))/2=f(2)/2
h=6
(f(3+6)-f(3))/6=f(6)/6
Если h меняется на 4, то и значение функции меняется на ту же величину, где тут скорость изменения у относительно х - не вижу.
>> No.136237 Reply
>>136206
> мимохудожник
Попробуй топологию, она предоставляет много простора для пространственно-образного мышления.
>> No.136241 Reply
>>136237
А что в ней пробовать и как? мне был любопытен лишь пример с гомоморфизмом объектов и некоторое время я определял из интереса какой простейшей фигуре гомоморфен тот или иной объект.
К тому же мне не обязательно художничество связывать как-то с матаном, мне интересны и инженерия и алгоритмы. Прост.
>> No.136247 Reply
Анон, кинь ссылку на какую-нибудь видео лекцию по интегралам, а то сам нахожу только бредятину с ютуба.
>> No.136258 Reply
Анон, в этом семестре у нас начинается математическая логика, и я прозреваю, что это дрочный предмет. Расскажи, что это такое и посоветуй хороший, годный, интересный учебник. Программа у нас стандартно-убогая, и за кучей подробностей не видно общей теории.
>> No.136263 Reply
>>136237
> Попробуй топологию, она предоставляет много простора для пространственно-образного мышления
Что в этой хуйне интересного кроме РОГАТЫХ СФЕР?
мимокрокодил
>> No.136292 Reply
>>136258
Общую теорию видно, Верещагин-Шень.
>> No.136339 Reply
>>136155
Решил всё-таки: 14!/4!10!=1001 способ выбрать 4 главы книги из 14. Распределяем их между 2 авторами:4!/2!2!=6. Теперь у нас 10 глав 10!/6!4!=210, 6!/3!3!=20. Последний автор выбирает 4 книги из 4, это 1. Перемножаем результаты, будет 25225200.

Что-то тредик умирает, где все, где тот няша с Гротендиком.
>> No.136340 Reply
>>136339
Какой няша? Спокойно, никто не умирает. Спешка ни к чему. Все заняты глубокими размышлениями.
>> No.136365 Reply
Анон, где применяется уравнение Эйлера? То, которое линейное однородное с переменными коэффициентами. Оно ведь не похоже на уравнение Эйлера из гидродинамики?
>> No.136382 Reply
В стране эльфов выводят классический анализ через нестандартный или нет? После его изучения стандартный кажется натужным и неудобным для изучения.
>> No.136384 Reply
Анон, как на тебя влияет алкоголь по твоим магическим способностям? Я просто на выходных распил аж 0.5 тёмного и сегодня на матане нихуя не понял, хотя обычно понимаю всё. Тема была ряды и всякие признаки вроде радикального коши или на чередующихся лейбница или на степенных абеля и т.п., от даламбера.
>> No.136385 Reply
>>136384
> магическим способностям
математическим
fix
>> No.136388 Reply
>>136384
> хотя обычно понимаю всё
Ой да ладно.
>> No.136392 Reply
>>136388
Вполне да вообще-то. Был задан вопрос о влиянии алкоголя на мозги, вместо ответа уже второй сказал какую-то хуйню.
>> No.136393 Reply
File: 090.PNG
Png, 92.85 KB, 840×534 - Click the image to expand
edit Find source with google Find source with iqdb
090.PNG
Почему k =<nx выбрано?
>> No.136404 Reply
>>136393
А что тебе не нравится? Это вполне логично. a, увеличенное на на 1, будет лежать между nx и ny для некоторого натурального n, разделив на n, получим, что (a+1)/n - рациональное, лежащее между x и y.
>> No.136408 Reply
>>136392
Если хочешь серьёзного ответа, то могу сказать, что если от 0.5 тёмного пива ты тупеешь и продолжаешь тупить и на следующий день, то пиво это димедрольное и пить такое я не рекомендую.
>> No.136414 Reply
>>136408
Это был козёл тёмный 0.5 за 63 рубля что ли. На вкус был каким-то сладковатым.
>> No.136417 Reply
>>136384
Уже давно не пью, но в институте собирались с пивком регулярно. На магические способности в том возрасте никак не влияло - порой даже домашку решали или лабы делали во время попойки...

Скорее всего, или тема сложная попалась или просто плохо выспался и внимание рассеялось. Почитай дома учебник/конспекты, разбери эту тему, да и всё.
>> No.136420 Reply
File: Фото0079.jpg
Jpg, 624.21 KB, 1536×2048
edit Find source with google Find source with iqdb
Фото0079.jpg
File: Фото0080.jpg
Jpg, 513.67 KB, 1536×2048
edit Find source with google Find source with iqdb
Фото0080.jpg
File: Фото0081.jpg
Jpg, 458.01 KB, 1536×2048
edit Find source with google Find source with iqdb
Фото0081.jpg
File: Фото0082.jpg
Jpg, 477.34 KB, 1536×2048
edit Find source with google Find source with iqdb
Фото0082.jpg

Аноны, как вообще называется представленное на фотках и по каким учебникам всё это легко можно выучить?
>> No.136422 Reply
>>136420
> как вообще называется представленное на фотках
Хуета хует.
> по каким учебникам
Демидович, Фихтенгольц.
> легко
Нет пути.
>> No.136423 Reply
>>136422
> Нет пути.
Бида, пойду страдать.
>> No.136425 Reply
Няши, реквестирую ясных гайдов по решению задач на метод Гринберга и задач Штурма-Лиувилля.
>> No.136426 Reply
File: кролик.PNG
Png, 134.09 KB, 930×894 - Click the image to expand
edit Find source with google Find source with iqdb
кролик.PNG
Как такое вообще выполнять? Нужно какие-то методы знать или как? Пока только первое сделал от противного, получил разность двух рац. чисел m-n=r` - иррациональное, что априори неверно как бы. Или надо и это доказывать теперь?
Всегда решали задания типа что-то посчитать. Может есть какой-то специальный задачник с такими вот заданиями по темам матанализа на доказательства, где они скомпанованы по нарастанию сложности?
>> No.136428 Reply
>>136426
Попробуй второе по индукции.
>> No.136447 Reply
>>136428
Нехорошо так шутить. Он же может голову сломать, пока поймет, что ты шутишь.

>>136426
Раз уж я раскритиковал шутку, то вот конструктив насчет второго: подумай, как построить требуемое иррациональное число. Например, придумай алгоритм, который бы выписывал это число.

Вообще, такие задачи решаются разглядыванием определений и уже известных простых свойств, пока не наступит просветление. Поэтому первым делом убедись, что все слова знакомые.

>>136420
Начни с первого параграфа Алгебры за 9-й класс Макарычева и Миндюка, что ли. А потом продвигайся по оглавлению. Это решит твои проблемы с последними двумя пиками, а возможно, и со вторым. Если не решит, бери Колмогорова за 10-11 класс.
>> No.136451 Reply
>>136426
Ну, во втором достаточно взять х=(r+r')/2
В третьем, показать что сумма максимальна, когда максимален первый элемент, а затем когда второй элемент.
А вообще, надо долго рпзглядывать определения и решать, секретного пути нет.
>> No.136452 Reply
>>136451
Перечитай условия.
>> No.136454 Reply
>>136428
Я даже не знаю что это.
>> No.136487 Reply
File: fourier_plot_mail...
Png, 119.91 KB, 1920×1920
edit Find source with google Find source with iqdb
fourier_plot_mail.ru-1.png
File: fourier_plot_dcm-...
Png, 160.17 KB, 1920×1920
edit Find source with google Find source with iqdb
fourier_plot_dcm-1.png
File: fourier_plot_sd-1...
Png, 98.14 KB, 1920×1920
edit Find source with google Find source with iqdb
fourier_plot_sd-1.png
File: fourier_plot_lj.c...
Png, 104.38 KB, 1920×1920
edit Find source with google Find source with iqdb
fourier_plot_lj.com-1.png
File: fourier_plot_5149...
Png, 80.61 KB, 1920×1920
edit Find source with google Find source with iqdb
fourier_plot_5149-1.png

Посоны, подскажите.
Есть временные ряды для двух классов, А (в приложенных примерах "sd", "dcm" и "5149") и Б ("lj.com", "mail.ru").
Класс А характеризуется наличием сильной периодической компоненты, класс Б довольно хаотичный, редкие "сильные" всплески и мелкая непериодическая активность (визуально это хорошо видно).
Я это различие хочу получить в численном виде, т.е. некую функцию P, которая P(A) давала бы что-то, близкое к 1, а P(Б) близкое к 0.
Пробовал зафигачить фурье (итоговые спектры там же на картинках), но что-то я не вижу там ничего, что можно было бы использовать для моих целей, особенно на "dcm", а мне он довольно критичен.

Какой матаппарат может тут помочь? Очевидные вещи типа взять длину интервалов и посчитать матожидание/сигму я проделал, но мне нужно чтобы учитывались и длина интервалов между всплесками, и длина всплесков, и амплитуда.
Местная профессура мне сказала посмотреть в сторону вейвлет-анализа, но или я тупой, или я что-то тоже не вижу там нужного метода.
>> No.136488 Reply
>>136487
Да, а еще чисто в порядке любопытства, откуда у "dcm" и "sd" взялись эти несимметричные точки в спектре?
>> No.136499 Reply
Чем отличается открытый интервал от закрытого?
   f(x) = f(a) для всех x ∈ (a, b) вот функции равны это вообще как?
>> No.136518 Reply
File: image.jpg
Jpg, 136.38 KB, 1428×304 - Click the image to expand
edit Find source with google Find source with iqdb
image.jpg
Алгебра Ленга, параграф про свободные группы. Что-то не понял, как добавление Z-ок поднимет мощность выше счётной? Вообще, как выглядят группы с несчётной мощностью?
>> No.136521 Reply
>>136518
> Что-то не понял, как добавление Z-ок поднимет мощность выше счётной?
Насколько я понимаю, речь идёт о мощности множества образующих? Прямое произведение n экземпляров Z имеет n образующих, прямое произведения конттинуума Z - континуум образующих. Добавь нужное прямое проиведение - всё будет.
> Вообще, как выглядят группы с несчётной мощностью?
Ну, скажем, возьми Z/2Z, помноженное на себя континуум раз.
>> No.136536 Reply
>>136521
Понятно.
>> No.136549 Reply
File: Screenshot-02062015-03:49:17-PM.png
Png, 280.42 KB, 956×491 - Click the image to expand
edit Find source with google Find source with iqdb
Screenshot-02062015-03:49:17-PM.png
Смотрю calculus one на курсере и не могу разобраться.
Почему, как бы так сказать, предел сужается только с положительной стороны? Х стремится к 1 как [1.1, 1.01, 1.001] и т.д, а не [0.9, 0.99, 0.999] ?
Если бы x стремился к +-бесконечности, то понятно с какого конца подходить.
>> No.136550 Reply
Анон, почему основная теорема алгебры так называется? Чем полиномы так важны для алгебры? Это же скучные функции.
>> No.136551 Reply
>>136549
Снимаю вопрос.
Что-то совсем не собраться.
>> No.136552 Reply
>>136550
Быдло потеряло два слова в названии теоремы. Настоящее название - "основная теорема алгебры многочленов C[x]"
>> No.136554 Reply
>>136552
Нихуя себе потеряли два слова. Многочлены это вообще малая часть алгебры
>> No.136555 Reply
>>136554
Всем пофиг.
>> No.136557 Reply
>>136549
> стремится
Значит имеет направление.
Видимо, направление по умолчанию положительно.
В языковом плане то же самое. Когда мы говорим, что икс лежит в некой области «от и до» — это означает, что «от минимума до максимума». Направление письма слева направо, в том числе числового ряда, направления кординатных линий и т.п.

А с третьей стороны вообще без разницы с какой стороны стремится. Важно, что в пределе он близок к значению настолько, насколько это возможно, то есть практически неотличим.
Допустим X у тебя стремится с другой стороны, то есть ближайшее число — это 1+(1/бесконечность). В знаменателе будет бесконечно малое положительное число вместо бесконечно малого отрицательного. Оба равно близки к нулю.
>> No.136558 Reply
Платиновый вопрос. Какой учебник можно почитать для восполнения школьной программы?
>> No.136560 Reply
>>136558
Мордкович - 7-8-9 классы, Колмогоров - 10-11
http://alleng.net/
По геометрии вот эти книги
http://rghost.ru/8f2wtsQgc
http://rghost.ru/8yNZm8ghL
>> No.136569 Reply
>>136550
Основная теорема алгебры, в сущности, утверждает, что для алгебраической замкнутости поля действительных чисел к нему достаточно добавить только один элемент. Это действительно удивительный факт, для многих других полей алгебраическое замыкание строится крайне сложно. Например, для p-адических чисел необходимо строить бесконечную последовательность расширений, а потом строить топологическое пополнение для построенного поля.
Важна она, например, тем, что большая часть алгебраической геометрии работает только над алгебраически замкнутыми полями. Не будь поле С алгебраически замкнутым, у нас не было бы и половины тех геометрических знаний, которые есть сейчас.
>> No.136573 Reply
никто не может помочь@спроси у анона

Где и как можно экономно и быстро достать в РФ книги по математике на английском? Интересуют геометрия и начальные курса алгема.
>> No.136576 Reply
>>136573
экономно и быстро и на английском нигде.
>> No.136577 Reply
Почему число Пи (то есть его цифры, не знаю как это написать) бесконечно?
Есть ли ноль и бесконечность? Как это понять?
>> No.136579 Reply
>>136577
Число пи — иррациональное число, то есть его значение не может быть точно выражено в виде дроби m/n, где m и n — целые числа. Но вполне может быть выражено в виде дроби с нецелыми числами. Впрочем это некрасиво и не принято.
> Есть ли ноль и бесконечность?
Нуль, это число обозначающее отсутствие числа. На подносе лежало ноль бананов. Фактически это некоторым образом виртуальное число введённое для удобства вычислений так же как и отрицательные числа.
В математике принято считать что бесконечность есть. На самом деле бесконечность довольно легко прифантазировать. Но доказать существование бесконечности невозможно. Невозможно опытным путём проверить существование бесконечности, для этого необходимо было бы бесконечное число времени ехать на Порше по бесконечной дороге, и то конца этому эксперименту не было бы.
>> No.136581 Reply
>>135911
(>>134538)
> Задача 10. Докажите, что следующие свойства отображения f:X → Y эквивалентны:
> 1) f - биекция;
> 2) f сюръективно и инъективно;
> 3) f обратимо, то есть существует такое отображение g:Y → X, что gf = Id(X), fg = Id(Y).
1, 2) Если f биективно, то действительно следующее утверждение:
(1) ∀x ∈ X, ∃y ∈ Y, f(x) = y ∧ (f(x) = f(x') ⇒ x = x');
Отображение f инъективно, когда ∀x, x' ∈ X, (f(x) = f(x') ⇒ x = x') (2);
Отображение f сюръективно, если ∀y ∈ Y, f⁻¹(y) ≠ ∅ (3).
Легко видеть, что (1) включает в себя свойства (2) и (3). След-но, если отображение биективно, то оно инъективно и сюъективно, равно как справедливо и обратное.

3) g = f⁻¹, если f удовлетворяет условию (1), то f, биективно. Если f биективно, то и обратное отображение f⁻¹ также биективно. Т.к. композиция биективных отображений f⁻¹ ∘ f также обладает свойством биективности, то и отображение Id(X):X → X биективно. Сказанное справедливо и для ff⁻¹ = Id(Y):Y → Y.
> Задача 11. Про каждые два из следующих множеств выясните, существует ли между ними биекция:
> а) множество натуральных чисел;
> б) множество чётных натуральных чисел;
> в) множество натуральных чисел без числа 3;
> г) множество целых чисел.
а, а) f:ℕ → ℕ = Id(ℕ) и, в общем-то, биективно всегда;

а, б) f:ℕ → ℕ(p) = {x ∈ ℕ(p) | x = 2n, n ∈ ℕ};
Данное отображение обладает свойством (1), так что f⁻¹ также биективно.

а, в) отображение f:ℕ → ℕ(Δ3) биективным не является, т.к. образ f[3] = ∅. Это значит, что |ℕ(Δ3)| < |ℕ|, ℕ(Δ3) ⊂ ℕ и если f⁻¹:ℕ(Δ3) → ℕ, то f⁻¹ ∘ f ≠ Id(ℕ).

а, г) f:ℕ → ℤ биективным не является, т.к. ℕ := ℤ \ {x ∈ ℤ | x ⩽ 0}, |ℤ| > |ℕ|. Т.е., ∃y ∈ ℤ, f⁻¹(y) = ∅.
---
ℕ(Δ3) - натуральные без тройки;
ℕ(p) - натуральные чётные.
>> No.136584 Reply
>>136569
> утверждает, что для алгебраической замкнутости поля действительных чисел к нему достаточно добавить только один элемент.
Из чего это следует? Сама теорема об этом ничего не говорит. Как ты это понял? Что читал длч того, чтобы это понять?
>> No.136585 Reply
>>136584
> Что читал длч того, чтобы это понять?
Читал формулировку теоремы. Формулировка теоремы: поле комплексных чисел алгебраически замкнуто. Поле комплексных чисел получается из поля действительных прибавлением единственного элемента - мнимой единицы.
>> No.136587 Reply
>>136576
А если я готов немного переплатить/подождать месяц? Только, естественно, пока цена хотя бы на уровне amazon'a.
>> No.136591 Reply
Анон, смотри какая штука есть: y'' - xy = 0. Решениями этого дифура являются функции Эйри.
Почему это уравнение решается так сложно? Я перерешал много похожих дифуров - они решались методами понижения порядка. Чем это уравнение особенное?
Есть какие-то общие методы определить, интегрируется уравнение в элементарных функциях или нет?
>> No.136597 Reply
>>136591
Ну, во-первых, откуда ты брал диффуры, чтобы их решать? Из задачника. А задачник составляется не с той целью, чтобы показать тебе все диффуры на свете, а затем, чтобы ты с его помощью тренировался применять методы решения диффуров, т.е. диффуров, которые бы не решашись сравнительно просто, в задачнике априори нет, ну или они присутствуют в качестве редких примеров.
Критерии разрешимости существуют только для некоторых классов уравнений и систем уравнений. Гугли "Критерий Луивилля-Мордухай-Болтовского" и книжку Хованского "топологическая теория Галуа".
>> No.136599 Reply
Так что, ни у кого никаких идей касательно >>136487 нет?
>> No.136600 Reply
>>136599
У меня нет, няша. Но ты немного подожди.
>> No.136611 Reply
Есть ряд измерений, у каждого два независимых параметра, т.е. <(a1,1; a1,2); (a2,1; a2,2); ...; (aN,1; aN,2);>
Мне нужно посчитать, насколько каждый из параметров разнообразен, а потом как-то свести эти два числа в одно. Есть мысль тупо взять s1 = stddev(a1,1; a2,1;...;aN,1); s2 = stddev(a1,2; a2,2;...;aN,2), нормализовать, а затем перемножить, но наверняка есть какой-то стандартный метод для решения такой задачи, не?
>> No.136618 Reply
>>136573
А зачем тебе это, если не секрет?
Большинство написанного до начала девяностых переведено на русский(переведено, как правило, отлично)и продаётся за копейки. Не говоря уж о том, что всё есть в интернетах.
>> No.136623 Reply
Анон, меня интересует, какие дифуры применяются в наше время. Линейные дифуры были актуальны во времена Эйлера, но наука продвинулась, а линейные дифуры все еще продолжают изучать. Зачем они? Где нужны? Сталкиваются ли математики с ними в реальных задачах? Что можно почитать про актуальность различных течений в теории дифуров?
>> No.136631 Reply
File: 14133854597360.jpg
Jpg, 1727.59 KB, 3072×2073 - Click the image to expand
edit Find source with google Find source with iqdb
14133854597360.jpg
>>136618
Видимо, я библиофил, потому что уж очень люблю аутентичное: букинистику всякую, книжки в оригинале и печатные.
>> No.136662 Reply
>>136631
Можешь попробовать печатать и переплетать сам.
Такой-то DIY и хендмейд
>> No.136664 Reply
>>136662
Можно объединиться с такими же библиофилами и открыть свою собственную типографию, работающую по принципу print on demand. В принципе, там не так уж и много денег потребуется.
>> No.136667 Reply
>>136447
> Начни с первого параграфа Алгебры за 9-й класс Макарычева и Миндюка, что ли. А потом продвигайся по оглавлению. Это решит твои проблемы с последними двумя пиками, а возможно, и со вторым. Если не решит, бери Колмогорова за 10-11 класс.
Пролистал оба учебника, ничего похожего не нашёл. Области определения функции заданы формулами, значения переменных, которые надо подставлять в формулы, нигде не указаны, так что, видимо, мой третий лист - это не то. У квадратичных функций переменные, внезапно, в квадрате, а у меня в степенях содомия, так что снова не то. Системы уравнений только на два уравнения и тоже без указанных иксов.
Как эта херня называется-то?
>> No.136708 Reply
>>133509
Не могу решить следующую задачу по теории вероятности, потому что условие кажется мне неполным. Непонятно, как именно людей рассаживают за столы. Здесь не указано, происходит это разом или последовательно. И ещё не пойму как считать - сначала вер-ть такой комбинации для первого стола, а потом для второго стола выборку из остатков? Что думаете, аноны?
Задание:
Трех мальчиков, трех девочек и трех взрослых посадили за два стола по три человека. Установите, с какой вероятностью за обоими столами оказалось по одному мальчику, одной девочке и одному взрослому?
>> No.136709 Reply
>>136708
В мешке девять шаров - три красных, три синих, три зелёных.
Из мешка вслепую вынули три шара, потом ещё три.
Установить, с какой вероятностью и в первый, и во второй раз были бы вынуты красный, синий и зелёный шар.
>> No.136724 Reply
>>136667
Вот ты же не прочитал первый параграф, а вместо этого начал возбухать.

ruwiki://Кусочно-заданная_функция
>> No.136732 Reply
Очень странно, что сколько не дифференцируй синус он либо +-косинус либо +-синус.
>> No.136737 Reply
File: логарифм.PNG
Png, 8.49 KB, 648×79 - Click the image to expand
edit Find source with google Find source with iqdb
логарифм.PNG
Почему принято считать, что a не равно единице? Ведь такой случай возможен, если и b единице равно. Может, дело в графике?
>> No.136739 Reply
>>136737
Потому что сколько бы раз ты не возводил единицу в степень, ты получишь единицу, очевидно же.
>> No.136743 Reply
>>136739
Понятно, конечно, что получим единицу. Но разве из этого следует вывод, что такие логарифмы с основанием 1 невозможны? Мой школьный препод именно так и говорит.
>> No.136744 Reply
File: Кандинский-2.jpg
Jpg, 85.74 KB, 627×896 - Click the image to expand
edit Find source with google Find source with iqdb
Кандинский-2.jpg
А геометрия-тред здеся есть?
Просто я понял, что хотел бы попробовать стать богом геометрии - с какой книги начать? В аттестате по ней 5, в то время как по алгебре 4, и всегда как-то с ней меньше проблем было.
А ещё я сейчас вроде как черчу на работе, да и вообще картинки интересней формул. Красивенькие такие фигурки там...
>> No.136749 Reply
>>136732
Это не странно, это определение синуса практически. Он везде всплывает как решение соответствующего диффура.
>> No.136750 Reply
>>136743
Потому что log1b не определено, если b не равно 1. Хотя бы потому, что logab = ln b / ln a , а натуральный логарифм 1 - ноль. На ноль делить нельзя.
>> No.136754 Reply
>>136743
Бессмысленные они.
Сам подумай, в какую степень нужно возвести 1, чтобы получить 1. В 2 или в 3?
А в какую возвести, чтобы получить 5? Найдёшь такую степень, получишь возможный логарифм.
>> No.136756 Reply
>>136744
Тебе бог дал аналитическую геометрию, бери, дрочи. Зачем тебе еще что-то?
>> No.136759 Reply
>>136756
Ну, если шишка встанет - буду и её дрочить, и ещё что-то.
Тогда порекомендуй пособие(я), которое помогло бы мозги настроить на чтение скажем так статей о текущем состоянии данной науки, пожалуйста?
>> No.136760 Reply
>>136744
> стать богом геометрии
Какой именно геометрии? Их много, enwiki://Glossary_of_areas_of_mathematics
>> No.136761 Reply
>>136737
Потому. Можешь доопределить.
>> No.136765 Reply
File: p6.jpg
Jpg, 18.09 KB, 436×211 - Click the image to expand
edit Find source with google Find source with iqdb
p6.jpg
>>136760
Всей! От простого (повторения школьной программы планиметрии и стереометрии) к сложному - Veni, vidi, vici
Геометрия для меня - как мир для Алексашки Македонского.
>> No.136766 Reply
>> No.136768 Reply
>> No.136786 Reply
Ряд тейлора для некоторых функций вроде e^x верен не для окрестности точки вокруг которой смотрим, а по всему R из-за того что функция бесконечно много раз дифференцируема и бесконечно непрерывна?


Password:

[ /tv/ /rf/ /vg/ /a/ /b/ /u/ /bo/ /fur/ /to/ /dt/ /cp/ /oe/ /bg/ /ve/ /r/ /mad/ /d/ /mu/ /cr/ /di/ /sw/ /hr/ /wh/ /lor/ /s/ /hau/ /slow/ /gf/ /vn/ /w/ /ma/ /azu/ /wn/ ] [ Main | Settings | Bookmarks | Music Player ]